<<

Number: 212 August 2020 BRIDGEJulian Pottage’s Double Dummy Problem

VER ANCE SE

♠ A 8 ♥ K Q 10 ♦ 6 5 4 3 2 ♣ J 10 2 ♠ K 10 7 ♠ J ♥ N ♥ 2 W E J 8 7 6 ♦ 9 8 7 S ♦ A Q J 10 ♣ A Q 9 5 4 3 ♣ K 8 7 6 ♠ Q 9 6 5 4 3 2 ♥ A 9 5 4 3 ♦ K ♣

Contract 5♠ by South Lead: ♥2

This Double Dummy problem can also be found on page 5 of this issue. The answer will be published on page 4 next month. of the audiences shown in immediately to keep my Bernard’s DVDs would put account safe. Of course that READERS’ their composition at 70% leads straight away to the female. When Bernard puts question: if I change my another bidding up on Mr Bridge password now, the in his YouTube what is to stop whoever session, the storm of answers originally hacked into LETTERS which suddenly hits the chat the website from doing stream comes mostly from so again and stealing DOUBLE DOSE: Part One gives the impression that women. There is nothing my new password? In recent weeks, some fans of subscriptions are expected wrong in having a retinue. More importantly, why Bernard Magee have taken to be as much charitable The number of occasions haven’t users been an enormous leap of faith. as they are commercial. in these sessions when warned of this data They have signed up for a By comparison, Andrew Bernard has resorted to his breach by Mr Bridge? website with very little idea Robson’s website charges expression “Partner, I’m I should add that I have of what it will look like, at £7.99 plus VAT per month — excited” has been thankfully 160 passwords according a ‘founder member’s’ rate that’s £9.59 in total — once small. Bernard knows his to Chrome, and the Mr of £11.99 per month. The you have chosen your bridge- core market, and there’s Bridge site is the only money will be taken from playing level. Both teachers nothing wrong in tweaking one on my list with a their accounts at midnight are highly personable your product to that market’s compromised password. on Sunday, and on Monday fellows, and maybe some needs. But as a man, I feel Gavin Wilson, as posted afternoon they will get their fans are sufficiently wealthy I’m not in the target segment. on the Mr Bridge forum. first glimpse of the site to subscribe to both. But By providing a three-times I can find no evidence of a behind the paywall. The if one is forced to make weekly bridge interlude data breach at Mr Bridge. more prudent customer who a choice, it is hard not be since the lockdown started, Chrome checks if the u/name chooses to inspect what they swayed by their own playing Bernard has provided an & password combination buy before they pay will have results: Andrew is one of admirable free service for has been compromised to stump up £13.99 a month. England’s top players, many of us. But that doesn’t in a data breach on some It’s not clear that the bridge whereas Bernard’s team mean that when the service is site or app. I have checked world needs yet another can’t even seem to win taken behind the paywall, we mine for this forum and it website. £11.99 is a lot of their casual Friday evening will all gladly fork out £11.99 has not been compromised. money — it is the price, matches on BBO. And a month. The interactivity on The data breach could have for example, of the most viewers need to know that Bernard’s free service has occurred on any site where expensive subscription there are almost religious been great, but the promise you have used the same to Netflix. There is no differences at the core of the that one of his helpers, rather combination in the past. contract, as far as I can Acols that the two men are than Bernard himself, will be Ruth Edmondson, see, which specifies what teaching: Andrew advocates live to answer questions in Mr Bridge website manager. the buyer is going to get. bidding 1NT with a 5-3-3-2 the subscribers’ forum for a Please note, you should It’s not at all clear what shape even if the five-card period each day is, relatively never use a password on will continue to be free, or suit is a major headed by speaking, not a great lure. more than one site. even whether the YouTube A-K, whereas Bernard says I suspect that, after Sunday sessions previously broadcast that with such a hand you midnight, when the monthly TRIPLE DOSE: Part One will disappear behind should open one of the fee has climbed even It is very good of you to keep the paywall. Disgruntled major. Another key systemic higher to £13.99, very, very the printed version of the customers who resort to legal difference: Andrew says you few people will subscribe. magazine going throughout process are going to have really should play the Jacoby Many potential customers the lockdown when the to rely on the evidence of 2NT convention, whereas could be permanently lost, other bridge magazines in the commitments Bernard Bernard still advocates the and that will be a great the UK (EBU and SBU) have made in his YouTube videos Delayed Game Raise. shame. I feel a re-think switched to online only. before the launch, together With no sign on the horizon on strategy is needed. with the blurb on the home of when face-to-face bridge The Readers’ Letters page has page of the new website. will return, there must be DOUBLE DOSE: Part Two been absent for two months (Yes, that page is accessible something more going on When I tried to sign on to and I wondered if the supply without a pre-payment.) in this service than viewers the Mr Bridge forum this had dried up. You did, some But that blurb is not simply trying to improve their morning, Chrome (my months ago, encourage us encouraging: the top bullet skills for a social game which browser), told me there to write more and I felt guilty on “Why become a Founder may never again be social. had been a data breach at that I had not done so. Member?” is “Support Even the most conservative Mr Bridge, and that I had Bernard Magee”, which estimate of the demographics to change my password Continued on page 4...u

Page 2 BRIDGE August 2020 Features this month include: ADVERTISERS’ BRIDGE 1 August Double Dummy Bridge Problem INDEX TheMr Bridge Monthly Magazine by Ryden Grange, Knaphill, 3 Clive Goff’s Stamps 2 Readers' Letters Surrey GU21 2TH 4 The Edwardian ( 01483 489961 4 July Double Dummy Bridge Problem Answer Tea Towel [email protected] by Julian Pottage 5 Bernard Mages's e-books www.mrbridge.co.uk 5 Mr Bridge 5 Severence shop: mrbridge.co.uk/shop Tea Towel 5 August Double Dummy Bridge Problem 12 2021 Luxury Diaries Publisher/Managing Editor by Julian Pottage 34 Bernard Magee's Mr Bridge Tutorial Software 6 Julian Pottage Answers Your Bridge Questions 34 Rules Associate Editor 7 Bidding Quiz by Bernard Magee Simplified Julian Pottage 39 Little Voice [email protected] 10 Slam Bidding. Part 1 by Andrew Kambites 43 Mr Bridge Luxury Playing Bridge Consultant 13 Slam Bidding. Part 1 Quiz Cards Bernard Magee by Andrew Kambites 44 Bernard Magee's bernardmagee Tutorial Software @mrbridge.co.uk 14 The Sheriff Pays The Price by 45 Bernard Magee Bridge 16 Simple Take-Out Doubles. Part 2 by 47 2021 Luxury Diaries Cartoons & Illustrations 49 Clive Goff's Stamps Marguerite Lihou 17 Declarer Play Quiz by David Huggett 52 Mr Bridge Fine Bone www.margueritelihou.co.uk 17 Defence Quiz by Julian Pottage China Mugs Technical Consultant 18 Simple Take-Out Doubles. Part 2 Quiz by Sally Brock Tony Gordon 19 Bridge Glossary – E by Julian Pottage REDUCE THE Typesetting COST OF YOUR 21 Bridge Glossary – F by Julian Pottage Jessica Galt POSTAGE [email protected] 26 Bridge Glossary – G by Julian Pottage British postage stamps Proof Reading Team 29 Bridge Glossary – H by Julian Pottage for sale at 90% of face- Catrina Shackleton value, all mint and with 31 Bridge Glossary – I by Julian Pottage Julian Pottage full gum. Mark Rixon 35 Slam Bidding. Part 1 Quiz Answers Quotations for Mike Orriel by Andrew Kambites commercial quantities Richard Wheen 36 David Stevenson Answers Your Bridge Questions are available on Clubs & Charities request. 37 Defence Quiz Answers by Julian Pottage Maggie Axtell Values supplied in [email protected] 39 Declarer Play Quiz Answers by David Huggett 100s, higher values 40 Catching Up by Sally Brock available, as well as 1st Events, Holidays & Cruises and 2nd class. ( 01483 489961 40 Seven Days by Sally Brock ( 020 8422 4906 Jessica Galt 43 Bidding Quiz Answers (1-3) by Bernard Magee [email protected] or 07719753767 Megan Riccio 44 Letter From Overseas by John Barr [email protected] 45 Bidding Quiz Answers (4-6) by Bernard Magee 8 clive.goff@london rugby.com Emily Hawkins 46 How Should Suit Quality Affect My Rebid? [email protected] by Julian Pottage Customer Services 47 Bidding Quiz Answers (7-9) by Bernard Magee Catrina Shackleton [email protected] 48 Sally's Slam Clinic 49 Bidding Quiz Answers (10-12) by Bernard Magee Printed in the UK by The Magazine 50 The Diaries Of Wendy Wensum Printing Company 51 Simple Take-Out Doubles. Part 2 Quiz Answers www.magprint.co.uk by Sally Brock

BRIDGE August 2020 Page 3 READERS’ LETTERS continued

t...Continued from page 2. “Bumblepuppy Days” by I therefore wrote a couple of Julian Laderman – which EDWARDI letters, one longish and one explains the evolution from HE AN short. I about to write whist to bridge. The term T another when I realised it “double dummy” originated ♠ 8 5 3 might be the opposite – you in the 1840s and describes ♥ Q 9 5 4 3 2 have probably had many, a game of whist with two ♦ 2 many emails wishing you players with two exposed ♣ A K 2 well and have not had the hands on the table. Since ♠ A 6 4 ♠ Void time or energy to decide all cards were known it was ♥ 6 N ♥ 10 8 7 which to publish if any. considered an excellent W E ♦ A Q 10 8 S ♦ K J 9 7 5 learning tool for the four ♣ 7 6 5 4 3 ♣ Q J 10 9 8 TRIPLE DOSE: Part Two handed whist game. ♠ K Q J 10 9 7 2 In the April issue of BRIDGE, ♥ A K J 208, an anonymous writer This book also answered ♦ 6 4 3 suggested that the answers another question that had ♣ Void to Julian Pottage’s excellent puzzled me for years. Many double dummy problems be of the classic coups and published in the same issue squeezes, like the Contract 5♠ by South as the question. This would , were discovered by Lead: ♥6 not be a good idea for it whist players. I was amazed would be too tempting to that they could visualise such look at the answer without complex plays in whist while trying very hard to solve only seeing and playing the problem. Anyway, if their own hand – for these 10. THE EDWARDIAN © Mr Bridge ( 01483 489961. Printed in the UK on 100% cotton. Available as a tea towel from Mr Bridge ( 01483 489961 you are really stumped, you plays are difficult enough can always get the solution in bridge while seeing and by inputting the hand into playing your own hand and THE EDWARDIAN “deepfinesse.com” which dummy. In fact these coups you can download free. As were usually devised and a matter of principle I never solved “double dummy” The Solution to July’s open Deep until I with all hands exposed. have solved the problem, then I check to ensure I’m However, as we appreciate Double Dummy Bridge Problem correct. However, since DF from Julian’s excellent can easily solve any DD problems, seeing all the 1. Play the ♥Q from dummy and the ♥J from hand problem, this also means cards does not detract from that prizes cannot be given. the difficulty of the problem 2. Cash the ♣A-K, discarding the ♥A-K or the skill of its creator. 3. a heart high, West doing best not to overruff The anonymous writer also 4. Give up a diamond says that the EBU magazine TRIPLE DOSE: Part Three has a policy of not giving Having retired after teaching 5. West wins and, to stop you from ruffing two quiz answers until the next bridge full time for 40 years, diamonds in dummy, cashes the ♠A and plays issue. In fairness to the I still maintain a keen interest another spade EBU, there are usually six in the game but mainly 6. You drop a high spade from hand on the first quizzes in English Bridge by computer. Hence I am round and win the second spade in dummy cheaply, and all but one (leads) have selling off my collection dropping the ♠7 under the ♠8 if West has led the ♠6 answers in the current issue. of 250 bridge books, 400 wallets with cards and 300 7. Ruff the third round of hearts high and cross back I had always wondered bridge magazines at very to dummy with a to score the long hearts for where the term “double reasonable prices (each item diamond discards dummy” came from as “with is priced individually). If you open cards” or “quadruple or any of your readers would Note that if you do not reduce to a void heart before dummy” seems more like to see the list, please losing the lead, West puts East in with a diamond appropriate. I found the email [email protected] and scores a heart ruff. n answer in the excellent book Ian Dalziel, by email. n

Page 4 BRIDGE August 2020 2021 BACK COVER Bernard Mr Bridge Limited Edition Fine Bone China Mugs Magee’s Bidding Quiz E-Books

£25 per set of four fine bone china mugs Next year's diaries are now Price includes delivery to UK mainland addresses ( 01483 489961 Now available in stock. All, I repeat, all early orders have been These unique bone china through the despatched. If you have not mugs were not originally Amazon got a 2020 diary, I still have intended for retail sale but some overs so one can be to be awarded to event bookshop. sent to you for no charge on winners on our overseas receipt of your 2021 order. holidays as well as our Easy to use with weekend events. one question Sold as a box set of four, per page with one of each as illustrated on answer overleaf. the back cover of this issue. £25 per box of four mugs, including postage. £3.99 each from amazon.co.uk

BACK COPIES

These diaries are, as usual,

£14.95 each. They come in three cover colours; ruby red, navy blue and bottle Number: 211 u 2020 green. They each have a BRIDGEJulian Pottage’s Double Dummy Problem useful ballpoint pen in E EDWARDIA TH N their spine. The club offer ♠ 8 5 3 ♥ Q 9 5 4 3 2 ♦ 2 ♣ A K 2 ♠ A 6 4 ♠ Void N RA ♥ 6 ♥ 10 8 7 W E E N ♦ S ♦ C promotion is a minimum of A Q 10 8 K J 9 7 5 V ♣ 7 6 5 4 3 ♣ Q J 10 9 8 E ♠ K Q J 10 9 7 2 E ♥ A K J S ♦ 6 4 3 10, with a mixture of cover ♣ Void colours of your choice, for ♠ A 8 Contract 5♠ by South £55 or pro rata. Lead: ♥6 ♥ K Q 10

This Double Dummy problem can also be found on page 5 of this issue. The answer will be published on page 4 next month. ♦ 6 5 4 3 2 GOFFIES STAMPS ♣ J 10 2 Rather than throw away ♠ K 10 7 ♠ J

any surplus copies of ♥ 2 N ♥ J 8 7 6 BRIDGE, I prefer to send ♦ W E ♦ 9 8 7 S A Q J 10 them to bridge clubs or ♣ A Q 9 5 4 3 ♣ K 8 7 6 groups in boxes of 20 or so. ♠ Q 9 6 5 4 3 2 ♥ A 9 5 4 3 I still have some overs, so if ♦ K you haven't already done so, ♣ Void drop [email protected] Clive Goff has advertised an email to that effect and a his surplus postage stamps box full will be dispatched Contract 5♠ by South at discounted prices in the by carrier. If over- ♥ pages of this magazine for subscribed, I will fulfill the Lead: 2 over 20 years. Indeed he is over-orders the following our most faithful advertiser. month. If you need some stamps at a discount, see his advert All good wishes 11. SEVERANCE © Mr Bridge ( 01483 489961. Printed in the UK on 100% cotton. Available as a tea towel from Mr Bridge ( 01483 489961 on page 3. Mr Bridge

BRIDGE August 2020 Page 5 Julian Pottage Answers Your Bridge Questions When Should I Break A Transfer?

In the June issue them will have the chance to South rejected leading a wrong, it would be normal of BRIDGE 210, in bid on the second round. major into declarer’s bid to make all thirteen tricks. Q Bernard Magee’s Apart from keeping the suits. I guess two declarers made excellent Bidding quiz, opponents quiet, establishing The choice seemed twelve tricks after South led in question 9, he says that the nine-card fit can help to be between leading a club but North, mistakenly partner should break (super with constructive bidding. through West’s strength thinking South had led accept) the transfer. I am Responder, if holding 9 or 10 in diamonds, or the unbid second highest from a poor not very happy at breaking points and a five-card suit, suit, clubs, which meant suit, put up the ace of clubs the transfer. Partner (when is not likely to be thinking leading away from a king. at trick one and switched responding 2♥ to a weak of game facing a weak 1NT South chose the latter, to a spade at trick two. 1NT) could have very few opener (or of doing so with enabling the defenders to Sorry I cannot readily points, with 5 or 6 spades, 6 or 7 if you play a strong cash the first two tricks; with explain how declarer and opponents could each no-trump). With a nine-card the rest of the cards well made only ten tricks. have 11 or 12 points and trump fit, however, game placed declarer made the no 5-card suit. You have values could well be present. remaining eleven tricks. ♣♦♥♠ opened, so opponents are The scores on the not strong enough to double ♣♦♥♠ traveller were: I am setting play or to bid at the two level. quizzes from your Twice, playing bridge on On the deal 1 x 4♥= -420 Q magazines for the computer, partner has below, do you 2 x 4♥+1 -450 fellow club members to try broken the transfer and each Q think East’s 2♠ 2 x 4♥+2 -480 and keep our skill levels time I went one down in 3♠. an overbid? I realise 4 x 4♥ +3 -510 up, but I am stumped by Peter Calviou by email. East-West would have Declarer Play Quiz from reached game anyway. How can this contract be BRIDGE 91, page 53, number While nobody limited to ten tricks? 4, which I found in your likes going down, Why do the overtricks online library. It says you A sometimes you will Dealer East. N/S Game. vary so much? can finesse trumps three get a better score going ♠ K 8 7 2 The Lodger. times and play the queen of down than allowing the ♥ 10 6 3 spades to discard your losing opponents to make a ♦ 10 8 5 An opener’s reverse diamond but if you play the contract. -50 or -100 is ♣ A J 4 should show extra queen on the third round better than -110 or -140. ♠ J 9 5 ♠ A Q 10 4 values whether East can ruff to take away N A If you have four-card ♥ K J 2 W E ♥ A 9 8 5 4 you are playing Acol or your discard; then as soon support for partner’s five- ♦ A Q 7 6 3 S ♦ K 4 some other system. This as you play a club you lose card suit, you know your ♣ Q 9 ♣ 6 5 overbid backfired because to the ace, in which case a side has a nine-card (or ♠ 6 3 it helped South to find diamond comes back and 6♥ longer) fit. If your side has a ♥ Q 7 the best lead, holding is down. What am I missing? nine-card fit, the other side ♦ J 9 2 declarer to eleven tricks. John Goodyear by email. is sure to have at least an ♣ K 10 8 7 3 2 If nobody has bid spades, eight-card fit and will quite South might lead the You ask about often have a longer fit. While doubleton spade. With the this deal in which the opponents have not West North East South king of spades under the A South is declarer in entered the auction yet, they 1♥ Pass ace, diamonds 3-3 and 6♥ on a diamond lead: may be planning to come in 2♦ Pass 2♠ Pass the hearts placed such at their next turn – both of 4♥ All Pass that declarer cannot go Continued on page 8 ... u

Page 6 BRIDGE August 2020 Bernard Magee’s Acol Bidding Quiz

This month we are dealing with disruption from the opponents. You are West in the auctions below, playing ‘Standard Acol’ with a weak no-trump (12-14 points) and four-card majors. It is your turn to call.

1. Dealer East. All. 4. Dealer East. Love All. 7. Dealer West. Love All. 10. Dealer West. Game All. ♠ 7 6 3 ♠ 4 3 2 ♠ 4 3 ♠ 6 4 3 ♥ A 3 2 N ♥ 4 3 N ♥ K 8 7 N ♥ K 8 7 N ♦ K 9 8 7 6 W E ♦ A Q 7 2 W E ♦ A K 3 W E ♦ K Q 5 3 2 W E S S S S ♣ 8 7 ♣ A J 6 5 ♣ A K 9 7 6 ♣ A 6

West North East South West North East South West North East South West North East South 1♠ 2♣ 1♥ 3♠ 1♣ 1♠ Dbl 2♠ 1NT Pass 2♦* 2♠ ? ? ? ? * transfer to hearts

2. Dealer East. Love All. 5. Dealer East. Game All. 8. Dealer West. Love All. 11. Dealer West. Game All. ♠ 7 6 ♠ 6 2 ♠ K 3 ♠ 6 5 ♥ 9 4 2 N ♥ K Q 9 8 4 3 2 N ♥ 8 7 N ♥ K Q 5 4 N ♦ A 7 6 5 W E ♦ 7 5 4 W E ♦ A 4 3 W E ♦ A J 2 W E S S S S ♣ A Q 9 2 ♣ 3 ♣ A K Q 8 7 6 ♣ A K Q 2

West North East South West North East South West North East South West North East South 1♠ 2♣ 1♣ 3♦ 1♣ 1♠ Dbl 2♠ 1♥ 1♠ 1NT 2♠ ? ? ? ?

3. Dealer East. Love All. 6. Dealer East. Love All. 9. Dealer West. Love All. 12. Dealer North. Game All. ♠ 7 6 ♠ A Q 8 7 6 ♠ 4 3 ♠ J 4 ♥ A Q 9 8 7 4 N ♥ A 9 7 5 4 N ♥ K 8 N ♥ 2 N W E W E W E W E ♦ ♦ ♦ ♦ 6 5 S 4 2 S K 8 3 2 S A 9 8 6 5 S ♣ 9 3 2 ♣ 3 ♣ A K 8 7 6 ♣ A K 9 4 2

West North East South West North East South West North East South West North East South 1♠ 2♣ 1♣ 3♦ 1♣ 1♠ Dbl 2♠ 3♥ 3♠ Pass ? ? ? ?

My Answers: My Answers: My Answers: My Answers:

1...... 4...... 7...... 10......

2...... 5...... 8...... 11......

3...... 6...... 9...... 12......

Answers on page 43 Answers on page 45 Answers on page 47 Answers on page 49

BRIDGE August 2020 Page 7 t ... Continued from page 6. at the table. Is it just a back, West allowing the ♠K force, in which case 3♣ case of having seen such to win. Next came a low club would be forcing because ♠ A K Q hands before or are top to the ♣10 and ♣Q. West the initial 2♣ response ♥ Q 7 6 5 players just geniuses? cashed the ♠A and played a showed game values. ♦ 8 7 5 Angela Buckley by email. fourth round. Given the fall ♣ Q J 7 of the ♠10 and with the aid ♣♦♥♠ ♠ 10 8 7 6 5 2 ♠ J 9 Glad to hear that of the marked third-round

♥ Void N ♥ J 10 8 2 you solved them club finesse (East-West were Please can I ask ♦ K Q J 9 W E ♦ 10 4 3 2 all. Red Balloons is playing a natural bidding for an expert S A ♣ 8 6 4 ♣ A 5 2 slightly different to some system), declarer now had Q opinion on a couple ♠ 4 3 of the problems because it nine tricks: three tricks in of hands from BBO? ♥ A K 9 4 3 involves a each suit apart from hearts. 1) Dealer on your right ♦ A 6 that you might not find in Paul Jones, opens: 2♥ (weak). I hold: ♣ K 10 9 3 the textbooks. Since we London. have not used any problems that require an advanced This is a good ♠ 8 7 You need to finesse twice technique with which some example of the need ♥ A K 9 2 in trumps rather than three readers will be unfamiliar, A to count points. South ♦ K 3 times – if you needed one like a , most has passed as dealer so was ♣ A K Q 10 5 more to dummy, that readers should be able unlikely to have as many as could indeed be a problem. to solve them given time. 12 points. The bidding and You get to dummy to take Players with good general the play to the first few tricks I bid 3NT, which was a the first finesse by crossing problem solving ability will made it very likely that South disaster as the lead was a to the queen of hearts. You solve them faster. Sometimes held the ♠Q-J and the ♣A. spade and partner had no get back to dummy with the looking afresh first thing The ♥A-J in addition to the stopper. What should I have first round of spades to take in the morning can help. ♠Q-J and the ♣A would give bid? I was obviously worried the second finesse. Only him 12 points. West could partner would blast to 4♠ after you have drawn East’s ♣♦♥♠ therefore be fairly confident if I doubled – should I just last trump do you play the East would have either the have accepted this risk? second and third rounds of Watching on BBO, ♥J or the ♥A, making it 2) Opponents are spades, thereby avoiding a I saw the defenders safe to switch to a heart. playing a strong NT; the diamond loser. Perhaps you Q miss several bidding starts as follows: spotted that the text does not opportunities to defeat a ♣♦♥♠ specifically mention playing a thin 3NT contract. Should West North East South fourth round of trumps after they have done better? I opened 1♥ with 1♦ 2♣ 2♦ taking the second finesse but a 4-5-3-1 shape ? the earlier text did specify Q and 11 points; drawing trumps, so playing ♠ K 5 partner responded 2♣, I I had: the fourth round is implied. ♥ 10 4 3 rebid 2♥ and partner went ♦ A K 5 3 3♣; would you take that as ♣♦♥♠ ♣ K 9 3 2 a or is that down ♠ K Q 8 4 ♠ A 7 3 2 ♠ 10 9 4 to partnership agreement? ♥ 9 7 5 2 I have just received ♥ K Q 8 2 N ♥ A 9 5 Martin Epstein by email. ♦ A W E the June issue ♦ J S ♦ 10 9 6 4 2 ♣ 9 8 3 2 Q of BRIDGE and ♣ Q J 8 4 ♣ 7 5 In Acol a simple rebid thankfully solved The ♠ Q J 8 6 by responder like Generator in two minutes. ♥ J 7 6 A that is not forcing. I doubled showing majors. Can I also say that I found ♦ Q 8 7 It is invitational but with a Is this correct usage of this the previous month's ‘Red ♣ A 10 6 minimum opening and a bid? Would I be better Balloons‘ challenging misfit you would clearly pass. just supporting partner’s indeed – it took a few hours With a stronger hand, clubs either directly to solve. The other one that West North East South partner starts with a strong or via a ? was nearly as challenging Pass jump shift or perhaps makes Michael Hunt by email. was ‘Protection‘. At least 1♣ Pass Pass 1NT a rebid that you cannot pass, when I look at this one Pass 3NT All Pass such as a reverse to 3♦. 1. Your 3NT again it comes back in Some pairs play a different seems reasonable. seconds so there is some West led the ♠2, the ♠Q system in which a two- A With 19 HCP and hope of playing such hands capturing the ♠9. Declarer over-one change of suit a strong 5-card suit, the correctly if they come up played a spade straight response creates a game hand is too good for a 2NT

Page 8 BRIDGE August 2020 overcall. The alternative is Holding five diamonds queen losing to the king. to double, planning to bid Dealer West. E/W Game. and knowing that a two- North returned a small 2NT if partner bids 2♠ – ♠ J 9 8 7 5 3 trick set was all South diamond taken in dummy by given how strong your hand ♥ Q J 7 5 needed to outscore those the jack, South discarding is, you would not expect ♦ Void making game, doubling the four and seven of partner to bid 4♠ very often. ♣ J 8 4 5♦ was clearly correct. hearts. Was this friendly?

From what you say, ♠ Void N ♠ A K Q 10 6 2 The wide range North- I led a small heart ♥ W E ♥ partner would have done 6 2 S 10 4 South play for the Norwegian off dummy, playing the something else and then ♦ A J 9 7 3 ♦ 10 5 2 2♥ opening must leave the king, losing to the ace. you would be bidding 3NT ♣ Q 10 9 7 6 2 ♣ A 5 partnership guessing at South resumed the on the next round anyway. ♠ 4 times – but it is awkward to attack on clubs with the 2. Partner might have five ♥ A K 9 8 3 play against for sure. I would king, which I ducked. I or six clubs and a four- ♦ K Q 8 6 4 play a narrower range myself took the ace of clubs when card major yet overcall ♣ K 3 but given the agreed range they were continued. 2♣ because the hand is then the 2♥ opening on the I led the queen of short in the other major, so North hand seems fine. hearts, felling the jack. doubling could work. Given West North East South I made two clubs, two your singleton diamond, it Pass 2♥1 Pass 4♥ ♣♦♥♠ diamonds, three hearts and is quite likely opener will 4NT2 Pass 5♦ Dbl3 one spade for eight tricks. bid 3♦ over your double All Pass Did we get the The scores were:- and then (after the likely 1 4-15 points, four hearts and five best defence on 2NT = +120 (twice), two passes) you will need spades. Q the deal below? 2NT-1 = -50 (three times). to bid 4♣. This is a minor 2 in the minors. We scored a shared top. Alex Mathers, factor, however, because 3 The vulnerability and good Northallerton. if your side has nine or ten diamonds led South to double. clubs and the opponents Dealer East. Love All. South’s lead from have nine or ten diamonds, When 5♦ doubled was ♠ K J 3 a five-card suit you are not going to want three down, North-South ♥ 8 A was normal. to sell out to 3♦ whatever scored 800 for 96.2%. ♦ K 10 5 4 North’s diamond return you do at your first turn. Name and address supplied. ♣ 10 9 5 4 2 was not just friendly – I If instead of doubling ♠ 10 7 6 4 2 ♠ A 9 suggest it was the result you bid 3♦, partner can This is a good ♥ 10 9 6 2 N ♥ K Q 5 3 of a misclick – or perhaps W E bid a 4-card major if illustration of just ♦ J 7 3 2 S ♦ A Q 8 6 North momentarily thought opener passes, so that is A how important the ♣ Void ♣ A J 8 you were in a suit contract a reasonable choice too. vulnerability can be. As it ♠ Q 8 5 and South could ruff. The hand is too good just was, losing 800 instead ♥ A J 7 4 If you play ordinary to raise to 3♣ and has too of a non-vulnerable game ♦ 9 Stayman, West should much defence for a pre- produced a terrible score ♣ K Q 7 6 3 probably bid 3♣ over 2NT emptive jump to 4♣. I do not for East-West. Had only planning to pass any reply. have a strong view between North-South been vulnerable If you play five-card double and 3♦. Possibly the instead, losing 500 instead West North East South Stayman, that is not so vulnerability and the scoring of a vulnerable game 2♣1 Pass sensible. I would just transfer method might tip the balance would show a profit. 2♦2 Pass 2NT1 All Pass into spades. You have given as also might knowledge of Since you need to escape 1 Benjamin, 20-22 points five scores for 2NT contracts. partner’s overcalling style. for one down for a 2 Waiting bid I assume that there were at unfavourable vulnerability, plenty of other contracts too ♣♦♥♠ it would take an extreme South led the six of clubs, and you have just not quoted hand to justify doing so. Even away from the king-queen. the scores. I find it incredible Do you think either placing East with a few useful Was this friendly? I won that every West player would North or West bid cards, West really could not with the jack, discarding pass 2NT when holding nine Q too aggressively expect to make 10 tricks. a diamond from dummy. cards in the majors and a on this deal from a local Reverse the vulnerability and I I set about diamonds void. n online bridge club? would agree with West’s 4NT. leading the ace then the

Email your questions for Julian to: [email protected] Please include your postal address

BRIDGE August 2020 Page 9 About Bidding Judgement by Andrew Kambites Slam Bidding Part 1

ake a close look at the following that partner sees it the same way. there were two problems with that. layout. I tried to adopt an intensely First, 4NT might not discover whether pragmatic solution to this problem, West was void in spades. Second, T which I called the ‘last bid’ principle. would 4NT be Blackwood? E/W Game. When the auction looks as if it might The players had agreed that 4NT in ♠ 3 ♠ 9 7 4 get complicated I fix in my mind which a competitive auction would be a way ♥ A Q J 10 4 3 N ♥ K 6 5 2 bid I would make if I had only one bid of investigating the best game contract W E ♦ Q 9 8 ♦ A K J 10 5 S left. If I am 100% sure I can conduct rather than Blackwood, maybe the ♣ K 7 3 ♣ A a scientific investigation without minor suits. So 4NT was no use. It risk of confusion I will continue to was very hard for East to admit it to investigate alternative final contracts. himself but North/South’s aggressive West North East South But all the time I have in my mind my pre-emptive bidding left East no 3♠ ‘last bid’ and if I ever have any doubt scientific way forward. He should have 4♥ 4♠ 5♣ All Pass that partner would correctly interpret jumped to his 'last bid': 6♥. my next call I would terminate the Psychology is rarely discussed in No, it isn’t a misprint. It occurred auction by making my ‘last bid’. Of bridge. It is incredibly hard for a player during a county A team match so the course this isn’t very scientific, but it is to admit that he doesn’t have the players were pretty strong. So how can intensely practical. I might not always means to investigate the right contract you explain this ridiculous auction? reach the perfect contract but I usually scientifically. It seems like admitting Quite simply, the players had a avoid stupid contracts. Bridge players defeat or in this case conceding that misunderstanding. Even good players need to understand that it is not always the opponents have got the better are not immune from this. East meant possible to reach the best contract and of you. I would suggest a general 5♣ to be a cue bid agreeing hearts, not aim for a perfection that doesn’t principle. showing the ♣A or a club void. West exist. This theme will occur a lot in If something isn’t agreed, beware thought it was natural. The subsequent this article and the one next month. of a misunderstanding. If it is agreed discussion was heated; where do you So what happened on the hand in a regular partnership then it think the fault lies? above? should be written down as part of In my opinion East should take East certainly had a 'last bid' system notes. the blame. It mattered not who was available: 6♥. If the bidding of North/ One of our Gloucestershire players is theoretically right. The point was that South made any sense then West could Keith Stanley who has won everything the meaning of 5♣ in this sequence not have more than a singleton spade, worth winning in English bridge. He had not been discussed, neither was and West was quite possibly void in has a saying: ‘If I don’t understand a it covered by any general agreement spades. East had his eyes on a grand bid, I pass.’ Keith’s partners learn to that the players had agreed. East slam. He wanted to cue bid the ♣A in avoid ambiguous bids. had invented a bid and just assumed the hope that West could cue bid 5♠. Now consider this layout. What do that West would understand. This He would then chance 7♥, assuming you think of East’s 6♦ bid? illustrates one of the most valuable West had the ♥A. The problem was lessons any bridge player can learn. that because he wanted 5♣ to be a cue Being able to put yourself in partner’s bid, he allowed wishful thinking to E/W Game. position is a skill most bridge players take over from realism. In my opinion, ♠ Q 6 ♠ Void find incredibly hard to master. a more realistic attitude would be this. ♥ K 8 3 N ♥ A 9 Just because you feel you are sure 6♥ is unlikely to be a bad contract. ♦ A Q 9 7 5 4 W E ♦ K J 10 3 2 S an undiscussed bid has a certain Perhaps 4NT would help to confirm ♣ 10 4 ♣ A J 7 5 3 2 meaning doesn’t necessarily mean that two aces were not missing but

Page 10 BRIDGE August 2020 West North East South or not there are any exceptions, but better than minimum values, the 1♦ Dbl 6♦ sense dictates that the fewer auction should be game forcing. exceptions you have the better. I will Opener can show extra strength 6♦ may not be very scientific but it now list some of the principles that are by jumping (Auction F), by reversing shows good appreciation as to the part of my system and are particularly (Auction G) or by rebidding no- value of the hand and hence is an helpful with high-level bidding. They trumps, showing a hand too strong to excellent ‘last bid’. It also has two very are written down as part of my system open 1NT (Auction H). Responder’s practical advantages. notes. main way of showing extra values is by First, it avoids complications and A new suit at the three level in an changing suit at the two level (all three potential misunderstandings. uncontested auction is game forcing. auctions). Second, the less bidding you do, the harder it is for your opponents to Auction A Auction B Auction F Auction G Auction H find the best . Although West East West East West East West East West East impressive scientific sequences make 1NT 2♦ 1♠ 1NT 1♥ 2♣ 1♥ 2♦ 1♥ 2♦ you feel good, you end up regretting 2♥ 3♦ 2♠ 3♣ 3♥ 2♠ 2NT all the bids you make in the auction except the last one because they tell In Auction A, 3♦ is a new suit at In traditional Acol, none of these opponents how to defend. the three level (2♦ was a transfer three rebids by West would have been It is quite possible here that North bid showing hearts, not diamonds). considered game forcing but, looked has ♣K-Q-9-8-6 and that if he leads 3♦ is game forcing. This gives the at logically, why would East want to the ♣K that will lead to a defensive partnership the room to agree trumps pass? The partnership clearly has the ruff at trick 1 with another club to cheaply and maybe make a cue bid combined values for game and if that lose later. I would suggest that if your below game level if East is interested is the case why not clarify matters by scientific bidding had shown partner in a slam. making them all game forcing. a spade void it would be far easier for Auction B shows the main type of Auction H is particularly interesting. North to find the ♣K lead. His double exception. The point is that East and West has 15+ points. East has at least makes it quite likely that he holds the West have both limited their hands: nine points and with a poor, misfitting ♠A-K and surely he will lead the ♠A East by responding 1NT (6-9 points) nine points he should have responded unless you persuade him otherwise. and West by rebidding 2♠ (showing 1NT. Yes, they may have just 24 points If you take the view (as I do) that no game interest opposite a 1NT and just occasionally might prefer to you are unlikely to find out enough response). 3♣ shows a long club suit in avoid game, but there are such huge to be sure whether the grand slam is a hand that was not strong enough to advantages of playing 2NT as 15-19 good, 6♦ has a lot going for it. This is respond 2♣. West is expected to pass. points and game forcing. In particular particularly so here because North/ A new suit at the four level after a it avoids the need for West to jump South undoubtedly have a good fit in one level opening bid is a cue bid or if to 3NT with 18-19 points, giving at least one of the major suits and can it is a double jump it is a . the partnership room to investigate be expected to pre-empt aggressively. alternative denominations below 3NT. Auction C Auction D Auction E If the last bid was no-trumps then Bidding Principles West East West East West East 4NT is quantitative. One way to avoid misunderstandings 1NT 2♥ 1♠ 2♦ 1♠ 2♦ is to acquire the ability to sense danger 2♠ 4♦ 3♠ 4♣ 3♠ 4♦ Auction J Auction K and avoid ambiguous bids. However, West East West East most partnerships don’t have the time In Auction C, 4♦ is a cue bid agreeing 1NT 4NT 1♥ 2♦ for exhaustive discussion, so most spades. East should have six spades 2NT 4NT club partnerships resign themselves to and the ♦A (or a diamond void) and short, sharp auctions? interest in a spade slam. It is normal to In Auctions J and K, 4NT is It might not be possible to discuss a cue bid the lowest first-round control quantitative. huge numbers of individual sequences so East is denying first-round control If the last bid in an uncontested but it is possible to build your system in clubs. auction was a conventional suit bid around a number of general principles In Auction D, 4♣ agrees spades as with no suit agreed then 4NT is that have application in a variety of trumps and shows the ♣A or a club quantitative. situations. Some of these principles void. can guide you as to whether a bid is My principle doesn’t apply to Auction L Auction M Auction N game forcing, forcing for one round, Auction E because 4♦ isn’t a new suit. West East West East West East invitational or not forcing. Others Remember, East bid a natural 2♦ on 1NT 2♣ 1♥ 1♠ 2NT 3♦ can guide you as to whether a bid is the first round of bidding. 2♦ 4NT 3♥ 4♣ 3♥ 4NT natural or conventional. Opener can open with 11 points. 4NT Once you have decided to adopt a Responder can respond with 6 points. principle you must decide whether If both partners show significantly u

BRIDGE August 2020 Page 11 t With Layout R you have 10 top tricks: hand in Layout R? In Auction L 2♣ is Stayman. 2♦ is the ♠A-K, ♥A-K-Q, two diamonds I raised this question in a seminar a conventional reply denying a 4-card once the missing ♦A is driven out on a recent bridge holiday. Most major so 4NT is quantitative. and the ♣A-K-Q. To make a slam guests thought along the lines of: In Auction M 4♣ is a cue bid you need two more. If hearts break ‘Who should ask for aces?’ They were agreeing hearts. Hearts are the agreed 3-2, the fourth heart will give you an surprised to find that was the last suit so despite the fact that 4♣ is not eleventh trick. If you are in 6NT, you thing on my mind. My thoughts were natural (a cue bid), 4NT is Blackwood additionally need clubs to break 3-3 (or as follows: with hearts the agreed trump suit. a squeeze) to give you a twelfth trick. As South, I know that we have two Auction N is interesting. West rebid However if you are in 6♥ the twelfth balanced hands with combined assets 3♥ because he was expected to do trick comes from ruffing a spade in of between 31-33 points. Thus slam so, not because he has hearts. Hearts your hand, thus making five trump is likely to be at best borderline and is East’s suit. So I think my principle tricks. 6♥ is an excellent contract, if that is the case if I am considering should mean that 4NT is quantitative, needing only a 3-2 heart break. 6NT is looking for a slam I want to find a fit. showing a hand worth a slam try an inferior contract, usually requiring If I cannot find a fit, I am prepared to opposite a 2NT opening bid and with a 3-2 heart break and either a 3-3 club give up on a slam. This is a matter of five hearts. 4NT suggests 5-3-3-2 shape break or a squeeze. judgement, not conventions. I have and maybe 11 or a poor 12 points. Of With Layout S, 6♥ is still an four hearts and four diamonds and a course you can agree otherwise with excellent contract, requiring only a 3-2 4-4 fit in either suit might make a slam your regular partner. trump break but 6NT is a much better good, however unless I am playing If the last bid in an uncontested contract. You have twelve top tricks the old fashioned Baron convention auction was a natural suit bid then on any distribution: the ♠A-K-Q, in response to 2NT (in which 3♣ asks 4NT is Blackwood. If you play key- ♥A-K-Q, three diamonds (once you opener to start bidding four-card suits card Blackwood then the last bid suit have driven out the ♦A) and ♣A-K- up the line) I probably don’t have the is the agreed trump suit. Q-J; indeed you have a trick to spare. means to investigate a diamond fit. The North hand is identical in Therefore I must bid 3♣ (Stayman, Auction P Auction Q Layouts R and S so could South have or 5-card Stayman) and if a 4-4 or West East West East realised during the bidding that he 5-4 heart fit comes to light then slam 1♥ 2♣ 1♥ 1♠ needed the 4-4 heart fit in Layout R might be possible. 2♥ 4NT 2♦ 4NT but the 4-4 heart fit would only be a In Layout R the auction will start hazard in Layout S? The answer is yes 2NT-(P)-3♣-(P)-3♥-(P)? What should In Auction P, 4NT is Blackwood, and these hands illustrate a general South do next? Is there anything agreeing hearts. principle: South can do to investigate that is not In Auction Q, 4NT is Blackwood Bidding balanced hands, 33 points is open to misunderstanding? Maybe agreeing diamonds. generally considered to be borderline 4NT, but would 4NT be your version Note that if you play Key Card when considering whether or not of Blackwood or quantitative? Are you Blackwood, after that it is essential to to bid a slam. If you are considering sure? Are you equally sure that partner know which is the agreed suit because a slam with balanced hands with will agree? If not, avoid it! How about the trump king is a key card. 33 points or fewer, finding a fit and 3♠, a cue bid. Are you 100% sure that playing in the suit contract may well partner will agree it is a cue bid? How Bidding slams with be necessary to give you a vital twelfth about a jump to 5♥, inviting partner balanced hands trick. If you have points to spare, 6NT to use his judgement as to whether or Looking at both hands, try to work out may well be the safer contract, giving not to raise to 6♥? That is my favourite the best contract with Layouts R and S. you more options. option, but might partner think it Suppose North opens 2NT (20-22 asks for something in particular, for points). How should South look at his example good trumps? Layout R Layout S ♠ A 3 2 ♠ A 3 2 ♥ A Q 6 2 ♥ A Q 6 2 Mr Bridge ♦ K 7 ♦ K 7 2021 diaries ♣ A K 6 4 ♣ A K 6 4 Only £14.95 each

N N or 10 for £55 & pro rata. W E W E S S With a ball-point pen and soft Kidrell ♠ K 10 ♠ K Q cover in a choice of colours; navy ♥ K 8 4 3 ♥ K 8 4 3 ♦ Q J 3 2 ♦ Q J 10 2 blue, ruby red, bottle green. ♣ Q 5 3 ♣ Q J 10 ( 01483 489961

Page 12 BRIDGE August 2020 Can you see the problem? It is very difficult doing something that is completely clear. And if that is Slam Bidding the case there is something to be said for the somewhat defeatist approach of not looking for a slam. Part 1 Quiz Incidentally, suppose the auction by Andrew Kambites with Layout R starts as in Auction T. (Answers on page 35) Should North raise to 6♥?

Auction T North South 1 Your partner opens 2NT (20-22 Auction 2J 2NT 3♣ points). Playing simple Acol what do West East 3♥ 5♥ you respond with Hands 1A, 1B and 1♥ 2♣ ? 1C? 2NT

My guests mainly took the view that North had shown 20-22 points by Hand 1A Hand 1B Hand 1C 3 Using my bidding principles, which of opening 2NT and having only 20 ♠ 9 6 3 2 ♠ A Q J 2 ♠ 9 6 3 2 the 4♦ bids in the following auctions points North was minimum, hence ♥ Q 5 4 ♥ Q 10 9 ♥ K Q 4 are cue bids? North should pass 5♥. I think this is ♦ Q 6 5 ♦ 8 7 ♦ Q 6 5 wrong. The point count undervalues ♣ A Q J ♣ Q 10 9 4 ♣ A Q J Auction 3A Auction 3B aces and kings and overvalues queens West East West East and jacks when you are considering a 1NT 2♣ 2NT 3♣ suit contract. The North hand has all 2♥ 4♦ 3♥ 4♦ of its points in aces and kings except For questions 2, 3 and 4 you might the ♥Q, a card that is bound to be a find it an instructive exercise to give valuable card for a 4-4 fit in 6♥. North the question to your regular partner(s) Auction 3C also has some shape: 4-4-3-2 shape to see whether they agree with you. West East being better than 4-3-3-3 shape. 2NT 3♦ If North had 22 points with lots of 3♥ 4♦ queens and jacks then some of those 2 Using my principles of bidding, are ‘quacks’ would surely be opposite the following auctions game forcing, shortage and hence represent forcing for one round, invitational or duplication of values. This 20 point a sign off? 4 Using my bidding principles, is North hand is better for a heart slam the 4NT bid in the auctions shown than many 22 point hands. Note Auction 2A Auction 2B quantitative or Blackwood? If it is that if North/South do succeed in West East West East Blackwood, what is the agreed trump bidding and making 6♥ it is down to 1♥ 1♠ 1♥ 2♣ suit? judgement, not conventions. Is this 2♥ 3♣ 2♥ 3♣ heresy? A slam bid with no mention of Blackwood. In my opinion, it would be quite Auction 2C Auction 2D Auction 4A Auction 4B reasonable for South, with no aces West East West East West East West East opposite a 2NT opening bid, to give up 1♥ 1NT 1♥ 1NT 2NT 4NT 2NT 3♣ on slam and just settle for game. South 2♥ 3♦ 2♦ 3♣ 3♦ 4NT would not necessarily score badly for this as inevitably there would be some pairs greedily looking for the extra 10 Auction 2E Auction 2F Auction 4C Auction 4D points in 6NT and going off. West East West East West East West East So how should South approach 1♥ 1♠ 1♥ 2♦ 2NT 3♣ 1NT 2♥ Layout S opposite a 2NT opening bid? 3♥ 4♦ 3♥ 4NT 3♠ 4NT This time South has 14 points. The combined total is 34-36 points and the two tens can only be good news. There Auction 2G Auction 2H Auction 4E Auction 4F are points to spare for a small slam, West East West East West East West East and no obvious intelligent way to find 1♦ 1♠ 1♥ 1♠ 1♠ 2♣ 1♠ 2♣ if a grand slam is good. South should 2♥ 3♣ 2♠ 4NT 2♦ 4NT raise 2NT to 6NT. n

BRIDGE August 2020 Page 13 Robin Hood's Bridge Adventures by David Bird The Sheriff Pays The Price

ith some reluctance, the The Sheriff led the king of hearts and managed to beat it!’ Sheriff raised the last Lord Edmond de Trières laid out his ‘Indeed, my Lady,’ the Sheriff replied. mouthful of bilberry dummy. The red-faced Hugh Bullard, ‘We always seem to do well together.’ Wcrumble to his lips. Lady Elaine de an expert player who was paid A few deals later, Hugh Bullard had Trières’ chef had excelled himself. handsomely to partner the noble lord, a chance to win the first rubber: The pike mousse had been delightful surveyed the dummy. Four diamond and the braised venison as excellent tricks would suffice for the contract. It as always. It was a feast that he would would be a good start to the evening’s Dealer South. Game All. remember fondly for a good time. play. ‘Well bid, my Lord,’ he said. ♠ 8 7 2 A price would have to be paid, he was Bullard held up the ace of hearts ♥ A J 8 3 2 aware ‒ a potentially tiresome couple until the third round, throwing a ♦ 9 7 4 of hours partnering Lady Elaine at the club from dummy. He then led the ♣ Q 5 bridge table. Still, it was not beyond diamond queen from his hand. ♠ J 5 3 ♠ K 10 9 6 4 N ♥ K 9 5 ♥ Q 10 6 his capabilities to offer fulsome praise, The Sheriff knew from the bidding W E every time she failed to give away a that declarer held the ace of diamonds. ♦ J 3 S ♦ K Q 8 6 2 trick. Meanwhile, he could console Lady Elaine might hold a doubleton ♣ J 10 6 4 2 ♣ Void himself with the thought of many such jack in the suit, but this would then ♠ A Q splendid meals in the future. fall on the second round. To beat ♥ 7 4 The four players left the oak- the contract, he might need to find ♦ A 10 5 panelled dining room and strolled declarer with the ♦A-Q-J. In that case, ♣ A K 9 8 7 3 to the cardroom, ornately decorated it would be possible to block the suit. in the French style. This was the first The Sheriff won the first round of deal: diamonds with his king and cashed a West North East South heart trick. What should he do next? The Lord Lady Hugh If he exited passively with the jack of Sheriff Edmond Elaine Bullard Dealer South. Love All. spades, declarer would simply unblock 1♣ ♠ 4 2 the ace-jack of diamonds and cross to Pass 1♥ 1♠ 2NT ♥ 10 6 dummy with the ace of clubs. Pass 3NT All Pass ♦ 10 8 5 3 2 The only hope was to dislodge ♣ A 9 5 2 dummy’s entry while the diamond The Sheriff led the ♠3 and Bullard won ♠ J 10 9 3 ♠ 8 6 5 suit was blocked. East’s king with the ace. He had seven ♥ K Q J 9 N ♥ 8 7 4 2 When the Sheriff switched to the tricks on top and would need at least W E ♦ K 9 7 S ♦ 6 4 king of clubs, Hugh Bullard raised two extra tricks from the club suit. ♣ K 10 ♣ J 8 6 3 an eyebrow. The Sheriff had imbibed What plan should he make? ♠ A K Q 7 copiously during the long dinner. How If the clubs broke 4-1, there would be ♥ A 5 3 could he still find such a devastating no problem. He could give up a trick in ♦ A Q J defence? the suit, setting up five winners there. ♣ Q 7 4 The defenders had scored four tricks The only risk was a 5-0 club break. Ah already, so declarer could not the yes, it seemed that a was club switch. He won with dummy’s available. West North East South ace and had to concede one down a Hugh Bullard led the ♣7 from his The Lord Lady Hugh few moments later. hand. When the Sheriff followed with Sheriff Edmond Elaine Bullard The ornately dressed Lady Elaine de the ♣2, he reached forward to play 2NT Trières was delighted, ‘I had only one dummy’s ♣5. If this card had lost to Pass 3NT All Pass jack in my hand,’ she said, ‘and still we East, the clubs could break no worse

Page 14 BRIDGE August 2020 than 4-1 and the game would be made. start of the next rubber, the Sheriff He ruffed the spade lead, dropping East showed out, in fact, but this consoled himself with a mouthful of a flamboyant ace from his hand. He caused no problem. rather splendid Spanish brandy. then finessed the trump queen, West Bullard crossed to dummy’s club showing out. queen and returned to his hand When the queen of diamonds with the queen of spades. The Sheriff Dealer South. Love All. was played next, Lady Elaine took unblocked the spade jack on this ♠ Void her ace and switched to the jack of trick and then won South’s ♣9 with ♥ 8 6 5 2 clubs. Declarer won with the ace and the ♣10. The defenders scored three ♦ K J 6 5 returned to dummy by ruffing the spade tricks, but declarer claimed the ♣ 10 8 5 4 2 king of spades. balance for his contract. ♠ Q J 10 9 8 3 ♠ 7 6 5 4 2 Hugh Bullard discarded his two ♥ Void N ♥ K 9 7 The Sheriff could recognise a W E potential club losers on the king and good piece of card play. ‘It makes no ♦ 10 9 4 3 S ♦ A 8 7 2 jack of diamonds. He was then able to difference if I cover the first club,’ he ♣ Q 9 3 ♣ J finesse the jack of trumps and play the explained to his partner. ‘Hugh would ♠ A K ace. ‘My hand is high,’ he said, facing win with dummy’s queen and set up ♥ A Q J 10 4 3 his remaining cards. the clubs, just the same.’ ♦ Q Lady Elaine emitted a deep sigh. ‘I Lady Elaine was not over-pleased to ♣ A K 7 6 had an automatic double on my hand,’ have lost the first rubber. ‘What did she declared. ‘I had to trust you. After you have in diamonds, partner?’ she a bid at the two-level, surely I can rely demanded. West North East South on you for at least one trick?’ ‘Jack doubleton,’ replied the Sheriff. The Lord Lady Hugh The Sheriff made no reply, Lady Elaine looked blackly across the Sheriff Edmond Elaine Bullard maintaining a neutral expression. Had table. ‘Lead the jack of diamonds and 2♣ the woman not asked herself why the I can set up the suit,’ she exclaimed. 2♠ Pass 4♠ 5♥ opponents had bid to the six-level? ‘There’s no way he would play clubs Pass 6♥ Dbl All Pass ‘I made my ace of diamonds,’ that way after a diamond lead.’ persisted Lady Elaine. ‘Add the trick It was only the memory of the The Sheriff overcalled the strong♣ 2 that I was expecting from you, and splendid dinner that restrained the opening, hoping to make the auction they would have gone one down.’ Sheriff from a caustic reply. Lead a more difficult for the opponents. When The Sheriff sensed he was on the diamond, when he held J-x-x in the Lady Elaine subsequently doubled verge of an inappropriate outburst. suit that she had overcalled? Was the a small slam in hearts, he reached He closed his eyes for a moment and woman totally mad? once more for his goblet of brandy. conjured the images, one by one, of ‘I would have led a diamond from How tiresome it was to partner such every course he had enjoyed earlier in your hand,’ Lady Elaine continued. a player. Did she really think he could the evening. ‘Hugh heard my spade bid and headed hold anything useful in defence, when In control of himself once more, he for 3NT. He must have been prepared the opponents had bid a slam? smiled at his partner. ‘Most players for a spade lead.’ The Sheriff led the queen of spades would have done the same on your It was the custom of the house to and down went the dummy. Hugh hand,’ he replied. ‘One down would take a small drink between rubbers. Bullard saw that he would need to pick then have given us a wonderful 100, As the cards were being dealt for the up the trump suit, at the very least. instead of just 50.’ n

BRIDGE August 2020 Page 15 About Doubles by Sally Brock Simple Take-Out Double Part 2 Responding With A Strong Hand

ast month we looked at what we Here, when you find a 4-4 need to make a take-out double fit it is your intention to raise to the Hand E of an opening bid of one of a suit, three level - inviting game. With a ♠ A 10 5 4 Land how we should respond to such a completely minimum take-out double, ♥ K 10 4 3 double. We make a simple response partner can pass but with a little extra ♦ 5 4 3 with up to 8 or 9 points, and with a few he bids on. ♣ 7 6 more we can make a jump response. After partner has doubled 1♣ or 1♦ Generally, that is all we need because it is unusual to have a game on when Hand C Hand D an opponent has opened the bidding. ♠ K Q 6 4 ♠ K 10 4 However, sometimes we have a hand ♥ 6 ♥ Q J 10 2 Hand F that is too strong, or too complex, for ♦ A 10 4 3 ♦ A 10 9 4 ♠ 6 5 4 a straightforward jump bid. Take a ♣ K 8 4 3 ♣ J 4 ♥ 8 3 look at the following hands after your ♦ A Q 5 4 LHO has opened 1♦ and partner has ♣ K J 6 2 doubled. Hand C is a great hand if partner has After partner has doubled 1♥ or 1♠ four spades, but if he has only three you might need to look further for a Hand A Hand B final denomination. Again, start with You could guess which suit to bid, but ♠ K Q J 4 ♠ K Q J 4 2♦. If partner bids 2♠ then there is no that seems a little silly. So the modern ♥ A K 10 3 ♥ A J 4 3 problem – you raise directly to game. treatment is to cue-bid with both of ♦ 7 3 2 ♦ 7 3 However, it is more likely that he will these hands. This cue-bid does not ♣ Q 5 ♣ 9 8 3 bid 2♥. Now you bid 2♠, showing four promise another bid. With a normal spades and forcing to suit agreement. weakish take-out double partner If partner also has four spades he responds in his longer major facing Hand A was the final problem from will raise to 3♠ with a minimum Hand E (which you will pass) and in last month. With plenty of values for (which you could pass if you had only his longer minor facing Hand F (again game, you could just jump to 4♥ or 4♠ invitational values) or 4♠ with a few you will pass). If partner has extras, – the problem is that partner's double extra values. If he doesn’t have four say 15 or more HCP, then he should does not guarantee four cards in both spades, he will either bid 2NT or bid a either make a jump responce or else majors, so you could end up in a 4-3 fit. club suit. return the cue-bid. The solution is to cue-bid opener’s suit, With Hand D you would like to Of course, if you start with a cue-bid 2♦. This cannot be natural because if invite game in hearts or no-trumps. and then bid a new suit, that is forcing you had a lot of diamonds, you would Again, bid 2♦. If he bids 2♥ raise him as before. simply pass the double. 2♦ is forcing to to 3♥; if he bids 2♠ bid an invitational suit agreement and asks partner to bid 2NT (you would bid 3NT with more). The subsequent auction his 4-card suits ‘up the line’. Here, if The above is the traditional partner bids two of a major you intend treatment of a cue-bid response to You have asked partner for their best to jump straight to game, knowing a double, and what I would assume suit and most of the time you will that it will be a 4-4 fit. without discussion. However, many raise to an appropriate level. After Hand B is similar but weaker. Now modern tournament players use it your RHO opens 1♥, you double and you are worth inviting game, but not slightly differently. Suppose you have partner responds 1♠, you hold: quite strong enough to bid it yourself. one of the following hands: Continued on page 18... u

Page 16 BRIDGE August 2020

DECLARER DEFENCE QUIZ

by Julian Pottage PLAY (Answers on page 37)

ou are East in the defensive positions below, playing QUIZ matchpoints with neither side vulnerable. While you Yusually aim to beat the contract, you may also need to by David Huggett consider the risk of conceding overtricks. (Answers on page 39)

ou are South as declarer playing or teams, 1. ♠ A K J 6 3. ♠ K Q 10 8 3 2 so aiming to make your contract. In each case what is ♥ A K Q 9 4 ♥ Q J 5 Yyour play strategy? ♦ 7 5 4 ♦ Q J 10 ♣ 8 ♣ 10 ♠ 8 5 3 ♠ J N ♥ 10 7 5 N ♥ A 10 7 1. ♠ J 7 3 2 3. ♠ A 7 5 W E ♦ J 6 3 W E ♦ K 9 8 4 3 2 S S ♥ K J 10 6 ♥ 6 5 ♣ A J 7 3 ♣ A J 7 ♦ 10 6 ♦ K Q 7 ♣ 9 6 5 ♣ A Q 7 6 5 West North East South West North East South N N 1♥ Pass 1♠ 1♠ 2♦ 2♥ W E W E Pass 4♣1 Pass 4♠ Pass 3♥ Pass 4♥ S S All Pass All Pass ♠ A K Q ♠ K Q J 2 1 Strong spade raise, short ♥ A 9 8 7 5 ♥ K 7 clubs ♦ K Q J ♦ A J 9 2 ♣ J 2 ♣ K 3 2 Partner leads the ♣2. What Partner leads the ♦5, won is your plan? by the ♦Q. At trick two you capture the ♥Q with the ♥A. Having opened 2NT fourth You are declarer in 6NT after What is your plan? in hand, you are declarer in West opened a weak 2♥. 4♥. West leads the ace, king West leads the ♣J. How do and a low club. How do you you plan the play? plan the play? 2. ♠ Q 4. ♠ K J 8 ♥ A Q ♥ 8 ♦ A K J 10 7 5 4 ♦ A K 10 9 6 4 2. ♠ K 9 7 5 4. ♠ K 7 5 ♣ 8 6 3 ♣ K Q 8 ♥ 8 6 ♥ A K 5 ♠ K 9 8 3 ♠ Q 10 9 2 ♦ A 5 4 ♦ K 9 7 5 N ♥ 10 7 5 3 N ♥ A 7 2 W E W E ♣ ♣ ♦ ♦ A 9 3 2 K 6 4 S 9 6 3 S Q 8 3 ♣ A 7 ♣ 7 5 3 N N W E W E S S West North East South West North East South ♠ A 4 2 ♠ Q 6 1♦ Pass 1♥ Pass ♥ K J 4 ♥ 8 Pass 3♦ Pass 3NT Pass 1♦ Pass 1♥ ♦ K J 9 6 2 ♦ A Q J 10 3 2 All Pass Pass 3♦ Pass 3NT ♣ K 5 ♣ A 9 7 5 All Pass

Partner leads the ♣Q. What Partner leads the ♣J. You are declarer in 3NT You are declarer in 6♦ after is your plan? Declarer wins in hand with after West has overcalled in West made a weak jump ♣ ♦ hearts. West leads the ♥7 overcall of 2♠. West leads the A and runs the J to and East plays the ♥10. How the ♥J. How do you plan the your ♦Q. What is your plan? do you plan the play? play?

BRIDGE August 2020 Page 17 t ... Continued from page 16. Hand L was too strong for a simple With Hand N you are very strong but 2♦ overcall, so double first and then you have no idea where you are going rebid 2♦. This shows in the region of to play the hand. Obviously if partner Hand G Hand H 18 HCP or so. It might have worked has five spades, 4♠ could be right; if ♠ Q J 5 4 ♠ K Q 4 3 better to overcall or rebid 1NT, but A-x he has something like ♥Q-10-x-x 3NT ♥ 6 5 ♥ 5 is not an ideal stopper unless partner could be right (your LHO probably ♦ A 7 6 3 ♦ A J 6 3 can help. does not have an entry); while if he has ♣ K Q 3 ♣ K J 6 5 neither of the above, then it could be right to play in a minor game. In order Hand M Hand N to keep your options open, follow Hand G is a very minimum take-out ♠ A 3 2 ♠ A J 4 partner’s 1♠ reply with a cue-bid of 2♥. double, both in terms of points and ♥ A ♥ 6 With nothing (and only necessarily distribution. There is no reason to do ♦ A J 4 ♦ A K J 4 3 four spades) partner should rebid 2♠ anything very much. Pass, for now, ♣ K Q J 10 7 6 ♣ A K Q 4 and then you rebid 3♦ showing a very preparing to bid 2♠ if the opponents strong hand and asking partner to bid bid again. something sensible (including pass). Hand H is better. You don’t have Hand M is very strong. Double and If partner has some bits and pieces he a real but you are certainly rebid 3♣. This is not forcing but shows should bid a new suit, or bid 2NT, or happy to compete to the two level so a very strong hand. Partner should bid jump in spades with a 5-card suit. you might as well do so now. Bid 2♠, something unless he/she has nothing Next month we will continue to look which you hope will silence everyone. at all. at the simple take-out double. n

Hand I Hand J ♠ A K 6 5 ♠ A K 6 5 Sally Brock's Simple Take-Out ♥ 7 4 ♥ 7 ♦ A Q J 6 ♦ A Q J 10 6 Double Part 2 Quiz ♣ A J 4 ♣ A J 4 (Answers on page 51)

What do you bid, as West, on the following hands with the Hand I is stronger: 19 HCP. You can auctions given? The vulnerability and scoring method should not see that game could be on if partner affect your answers. has 5 or 6 points. Jump to 3♠ to tell him so. Hand J is the same in terms of high-card points but has much more Hand 1 Hand 2 Hand 5 Hand 6 playing strength because of the fifth ♠ K Q 5 4 ♠ 5 4 ♠ A Q 10 ♠ 5 diamond and the singleton heart. If ♥ A J 10 2 ♥ A 3 2 ♥ K 10 3 ♥ K Q J 10 3 partner has as little as Qxxx in spades, ♦ 8 7 ♦ K 5 ♦ A J 4 ♦ A J 4 there is a reasonable chance of game, ♣ Q 4 3 ♣ A Q J 10 5 4 ♣ A J 10 4 ♣ A K 3 2 so bid 4♠. Sometimes you have a strong hand without four-card support for partner. West North East South West North East South Again, your RHO has opened 1♦ Dbl Pass 1♠ 1♥, you have doubled and partner bids ? Dbl Pass 2♣ Pass 1♠. You hold: ?

Hand K Hand L Hand 3 Hand 4 Hand 7 Hand 8 ♠ A J 3 ♠ Q 10 4 ♠ A 10 9 4 ♠ 9 8 7 3 ♠ 5 ♠ 5 4 ♥ K Q 5 ♥ A 4 ♥ K 4 ♥ K Q J 10 5 4 ♥ A Q 4 3 ♥ A K 3 2 ♦ A Q J 4 ♦ A K J 6 5 ♦ Q J 10 4 ♦ K 5 ♦ A J 4 3 ♦ A K 6 2 ♣ Q J 4 ♣ A 6 5 ♣ J 5 3 ♣ 4 ♣ K Q 5 4 ♣ A K 5

With your 20 HCP on Hand K you West North East South West North East South were too strong to overcall 1NT, so 1♠ Dbl Pass 1♠ you double and then rebid 1NT. That ? Dbl Pass 2♣ Pass shows in the region of 18-20 HCP and ? partner can bid accordingly.

Page 18 BRIDGE August 2020 A to Z of Bridge A to Z of Bridge compiled by Julian Pottage

and draw trumps. You want to make the opponents lead hearts, saving you The regulatory body for Duplicate from having to guess who holds the Bridge in England. It keeps master- E queen. point records for its members and Before giving up the lead, you must organises National tournaments in play three rounds of clubs, ruffing the England. It has a headquarters in third, to eliminate the suit. Then, when Aylesbury and a staff of 16 including EAST you concede a spade, the defenders part-timers. One of the positions at the bridge table. will have no safe exit. ENTRY ECHO ENCOURAGING A card you can use to enter a particular The play by a defender of an A term applied to a bid or a card that hand. In most cases, an entry is a unnecessarily high card on the first urges partner to bid on or to continue winner facing a low card, though round of a suit followed by a lower leading a suit. Any bid that is not a sometimes you can create an entry by card on the second round is an ‘echo’ sign off, limit bid or a pre-emptive bid overtaking a high card or by ruffing. (eg the seven followed by the two). In is likely to be encouraging. Some limit Entries are important for declarer the UK, the more common term is a bids are encouraging too. and the defenders. The need to retain ‘peter’ or ‘high-low’. You usually use it entries means that it is rarely right as a to partner that you like the to begin the play by cashing all your suit, or that you have an even number A term used when an opponent is side’s top winners. of cards in it. given the lead at a vital stage of the play and, with his subsequent lead, ENTRY KILLING PLAY ELIMINATION is forced to concede a trick or tricks A defensive manoeuvre designed to The process of removing neutral and is therefore ‘endplayed’. The vital destroy entries either in declarer’s hand, cards from defenders’ hands so that point is more likely to occur in the end or in dummy. This may be via a lead or they have no safe exit when they are stage of the play, hence the name. For by timing when you cover an honour. thrown in (given the lead). You can example: bring about elimination by cashing winners or ruffing losers. Q J 10 6 ♥ 9 N ♦ A 6 4 W E K 7 5 2 ♠ 7 5 S

♥ A J 6 ♥ Q N ♥ 8 A 9 8 3 ♦ K 10 7 4 2 ♦ K J W E ♦ 10 8 S ♣ K 8 5 ♥ J East, if trying to deny declarer a N ♦ Q 4 second entry, should not cover any of W E S dummy’s honours, instead saving the king to play on the six. This way, the ♠ A 10 South, on play in no-trumps, needs ace is an entry but the nine is not. ♥ K 10 2 two of the last three tricks. ♦ A Q 9 8 6 3 If he leads a diamond, West will EQUALS ♣ A 9 win two tricks. Instead, he uses an A holding of cards (in the same ‘endplay’ by putting West on lead hand and suit) in sequence at the with the ♥Q. West is now ‘endplayed’, start of play, or of the same value as As South, playing in 6♦, you receive forced to lead diamonds, conceding the play progresses, is a holding of a spade lead. You win with the ace the last two tricks. ‘equals’. u

BRIDGE August 2020 Page 19 t For example, at the start of play If West leads low and East takes the ten any scoring errors. the queen and jack in a given suit and with the ace, South can drop the king For example, if you think you have in the same hand are equals; once to leave the queen as an established had a 55% session and the posted score the jack has gone, the queen and ten entry. shows you with 50%, it may be worth become equals. your while to check the scores very ESTABLISHED carefully for possible errors. ESCAPE MECHANISM A revoke becomes established (i.e. it A bidding method used to locate the cannot be corrected) when one of the ETHICS safest fit after the opponents have offending side leads or plays to the The Proprieties of the game now form doubled for penalties. next trick. A revoke on the twelfth part of the actual Laws. Often applied after the double of trick never becomes established. Bridge is a game in which players a weak or mini no-trump opening. Penalties apply to and one should make an active effort to play in For instance, some play that after should try hard to avoid making them. an ethical way. As far as you can, try 1NT-double, responder redoubles If partner shows out of a suit, you to make each bid and play in an even with a single suited hand (and shows may enquire ‘having none?’ to check . Avoid displaying any emotion the suit after opener’s forced ♣2 ) that the play is not a revoke. This whether the play of the cards is more while an immediate bid after the applies whether you are a defender or or less favourable than you expect. double would show the lower of two dummy. Personal Ethics, also known as suits. An SOS redouble is another Active Ethics, are the efforts that example. ESTABLISHED SUIT many players make beyond what the A suit where a partnership holds all laws require. For example, you are ESCAPE SUIT the remaining high cards. not required to point out your own A suit held in reserve by a player revokes; many players, in fact, do so making a lead-directing or . because their own ethical approach, J 10 6 i.e. their Personal Ethics, require it. West North East South N 3♥ 7 W E Void ETIQUETTE 3♠ 4♥ 4NT 5♦ S In general, good manners at the bridge Q 8 table. (For example, declarer thanks South wants a diamond lead against a partner when dummy appears). spade contract and intends to retreat Be courteous to your opponents to hearts, the escape suit, if doubled in With the ace and king having gone, – remember they want to enjoy 5♦. A gambling 3NT opening bid is a this is an established suit for N/S. the game too – and avoid taking a systemic bid with an escape suit. disproportionate amount of the time ESTIMATE YOUR SCORE on your own bids and plays. ESTABLISHED CARD Some players like to record on their If you play more slowly than anyone A card that has become a winner. scorecards an expected number of else does, this disrupts the movement matchpoints or percentage on each and affects everyone’s enjoyment of as they play them. the game. Q 7 6 In the early rounds, you will base N your estimate largely on general EVEN W E S bridge knowledge and the difficulty or 1 A term applied to an even division J 10 otherwise of achieving the result. of the outstanding cards (3-3 As the rounds progress, the other is the even break with six scores on the traveller will play a cards outstanding; 4-3 an even After the ace and king have gone, the greater part in the estimate. distribution of seven). queen will be an established card. Different methods are in use for 2 An even number of cards in a suit estimating your score. Some players (2, 4, 6 etc.). ESTABLISHED ENTRY score on a scale of 0-4; 0 for a bottom, 3 An even card is one with an even A card that has become a winner and 1 for average minus, 2 for average, 3 number of pips (2, 4, 6 etc.). that faces a low card for leading to it. for average plus and 4 for a top. Others use a scale of 1-10. EXHAUST One advantage of estimating your To draw all the outstanding cards in Q 10 score is that towards the end of the a (trump) suit from a particular hand. event, if you think your score is not N J 8 4 3 2 W E A 9 7 6 good enough, you can vary your EXIT S tactics to obtain abnormal scores. To surrender the lead. The term K 5 Another advantage is that knowing usually applies when you do not want your estimated score can highlight the lead. For example:

Page 20 BRIDGE August 2020 In this side suit, West might drop the ♥ 5 4 queen under the ace. This may dissuade declarer from playing a second round N ♥ Q 10 9 6 W E ♥ Void while any trumps are still out. S F ♥ A K J 3 FALSE PREFERENCE After partner has bid two suits, FACE responder may choose to play in the You cash the ace of hearts and find To turn a card so that its front is visible first suit bid despite the fact that he is East now void. You then ‘exit’ with to the other players. Unless you are the longer in the second. a low heart leaving West to return a dummy, facing a card usually means heart into your tenace. that you are playing it. West North East South 1♥ Pass EXIT CARD FACE-TO-FACE BRIDGE 1♠ Pass 2♦ Pass A card used to exit in the hope of a Bridge played at a bridge table, where 2♥ favourable return or a safe card with you can see and or hear the other which to exit. In the above example, players, as opposed to online bridge, your exit card is the three of hearts. where players connect via computer. ♠ A 10 8 5 ♥ Q 6 EXPLANATION OF A CALL FACED LEAD ♦ Q 10 3 If an opponent asks you to explain In duplicate bridge, the opening ♣ 9 7 5 3 a call that your partner has made, lead must be made face down. This you should explain the partnership is to allow partner to ask questions agreement fully according to the regarding the auction without West bids 2♥ rather than passing 2♦ system you are playing. It is wrong to influencing the choice of lead. When for several reasons: say what the call ought to mean, or to these questions and replies are over, 1 If East has a good hand (one almost say how one proposes to interpret it. the leader turns the lead over to make worth 3♦ over 1NT), game may be If the partnership has no agreement it ‘faced’. on; (explicit or implicit) then you should 2 If East has more hearts than say so. You should not offer an FACTORING diamonds, as is certainly possible, a explanation unless opponents ask. The process of adjusting a matchpoint 5-2 fit may play better than a 4-3 fit; score. If, for example, some pairs have 3 At matchpoints, if both 2♦ and 2♥ EXPOSED CARD played fewer boards than the rest of are making, the difference between A card wrongly or inadvertently the field then the scorer must factor 110 and 90 may be crucial. exposed during the auction or play. up their scores by the appropriate Penalties apply for cards exposed fraction. Alternatively, the scorer may FARRINGTON, FRANK during the auction or by a defender. factor down the scores of the pairs Author of the first book of Duplicate playing more boards. Expressing Bridge Movements, once considered EXPOSED HAND scores as a percentage (matchpoints the definitive work on the subject. 1 The dummy is the exposed hand scored compared to matchpoints (as opposed to declarer, the closed available) is another way of factoring FEATURE hand). the scores. A holding in a suit, such as the ace, 2 Declarer or a defender may expose king, or maybe the queen, that is likely their hand when making a claim. FALL OF THE CARDS to be important on the given hand. The disposition of the cards as If you and your partner agree a EXTENDED STAYMAN ordained by fate. below the level of 3NT, it is After a ♦2 response to the 2♣ enquiry, usual to play that bidding a new suit a further enquiry bid of 3♦ asks about FALSE CARDING shows a feature (stopper) rather than three-card major suits. In reply, 3♥ Deceptive play of a card in the hope length in the suit; eg 1♦-3♦-3♠ shows shows three hearts while 3♠ shows that the opposition will misread the strength in spades (and by inference three spades. holding in the suit. weakness in hearts). Some people play that opener has two further options, to bid 3NT with FIELD 2-2 in the majors or 4♣ with 3-3 in the A K 10 6 4 2 In a pairs’ event, the field is the players majors. at the other tables, in other words your N Q 3 W E J 8 7 competitors. You will often see the S word field preceded by an adjective 9 5 such as strong or weak to describe the strength of the competition. u

BRIDGE August 2020 Page 21 t lower ranking card when leading his partner’s short suits. FIELDING A PSYCHE towards a tenace. For example: If a player makes a psychic bid and his FIT NON JUMP partner takes a subsequent action that This is a bid showing values in the illegally takes into account that the A Q suit bid as well as support for partner. first player has psyched, he is ‘fielding’ It usually occurs after the bidder has N the psyche. To allow for a psyche is not K 7 6 5 W E J 10 9 8 4 already passed or taken some other in itself illegal, either if the subsequent S action that precludes a purely natural auction has exposed partner’s psyche, 3 2 interpretation of the bid. or if the call that allows for the psyche is the normal action anyway. To ‘field’ West North East South a psyche before it has been exposed by If West has the king, this suit will yield Pass 1♠ 2♥ 3♠ an abnormal action is illegal. two tricks if South leads towards the 4♣ tenace, playing the queen unless the Auction 1 king appears. A finesse position arises Having turned down the easy chance West North East South in a number of different forms, but all to bid some number of clubs as dealer, 1♣ Dbl 1♠ Pass of them depend on the assumption West can hardly mean 4♣ as showing 2♣ that a particular missing card or cards just clubs. It shows values in clubs and lie in a certain hand. support for hearts. Auction 2 If North passes over 4♣, East West North East South FIRST IN HAND normally corrects 4♣ to 4♥. If North 1♥ Pass 2♦ Pass The dealer, the first player to have the bids 4♠, East can compete further if Pass opportunity to open the bidding. holding a club fit or lead a club against 4♠ if not. In auction 1, if West has four spades and East does not, West has fielded the A rare convention that uses a bid of FIT SHOWING JUMP (FIT JUMP) psyche by failing to support spades. In the next higher-ranking suit after This is a single jump, in a competitive auction 2, West’s pass of the forcing an opposing three-level pre-emptive auction or by a passed hand, showing response exposes the psyche. East can opening bid as a take-out request (eg length and strength in the suit bid and legally proceed on the assumption that 3♥-3♠). Sometimes wrongly called primary support for partner (nine or West has psyched. ‘Herbert’. The hand will normally more cards between the two suits). It contain at least four cards in the suit is forcing for one round. FIFTH ACE bid. As the trump king is often as important West North East South as an ace, especially when it comes to FISHBOWL Pass Pass 1♥ Pass bidding a slam, some conventions Method of allowing an audience to 3♣ (such as Keycard Blackwood) treat the view tournament bridge. The players king of trumps as the ‘fifth’ ace. sit inside a large glass surround, West North East South enabling the spectators to watch 1♥ 1♠ FILM without disturbing the players. The 3♣ A rare conventional defence to an use of a fishbowl is now rare. opening pre-emptive three-level bid. In the first auction, 3♣ cannot be a The acronym stands forFI shbein and FISHING CLUB strong jump shift because West passed Lower Minor. Over an opening 3♦ or This ♣1 opening does not promise as dealer. In the second auction, many 3♥ bid, Fishbein (the next suit up) is a length in the suit – typically used with pairs are happy to give up on the take-out request guaranteeing at least a mini or a weak no-trump. The idea strong jump shift after the overcall four cards in the major you bid, while is that by opening 1♣ you can be sure and play this as a fit-showing jump. In 4♣ is for take-out denying four cards of being able to rebid 1NT over any either auction, West might hold: in that major. Over an opening 3♣ or change-of-suit response. The fishing 3♠ bid, the lower minor is a take-out club differs from ‘better minor’ in that request. only 1♣ is a prepared bid. ♠ 7 5 ♥ K 8 5 3 FINAL BID FIT ♦ 7 4 The last bid, normally the bid before 1 The combined holding in a suit eg ♣ A Q 10 6 3 three consecutive passes end the K-8-7-3 facing Q-J-9-2 is a 4-4 fit. auction. 2 The two hands of a partnership are said to fit well if, for instance, One benefit of the fit-showing jump FINESSE one player has high cards in his is that it enables partner to judge An attempt to win a trick with the partner’s long suits and low cards in whether a two-suit fit is present.

Page 22 BRIDGE August 2020 FIVE ACE BLACKWOOD FLAT BOARD A version of Blackwood in which the ♠ A Q 9 5 3 ♠ K 10 4 In duplicate pairs, a deal on which all ♥ A K 4 N ♥ Q 8 2 king of the agreed trump suit counts W E the contestants record the same result. as a fifth ace, also known as Keycard ♦ J 7 4 S ♦ 8 3 In teams, a deal on which both sides Blackwood. In response to 4NT: ♣ A K ♣ J 8 6 3 2 record the same score or a score that differs by only 10 is flat. 5♣ Shows zero or 5♦ Shows one or five aces West East FLAT HAND 5♥ Shows two aces 2NT 3♣ A , particularly the 5♠ Shows three aces 3♠ 4♠ 4-3-3-3 pattern.

Except that 5♦ might show five aces, FIVE OF A MAJOR OPENING FLINT CONVENTION the responses are the same as those to This very rare opening shows an A convention invented by the English normal Blackwood. eleven playing-trick hand missing player, Jeremy Flint, designed to allow both top trump honours. Responder the partnership to play in three of a FIVE-CARD MAJORS passes with neither top honour, raises major after an opening bid of 2NT. Some systems require that you need to six with one of the two top honours, Immediate responses of 3♥ and 3♠ are a five-card suit to open ♥1 or 1♠. and to seven with tboth top honours. natural and forcing, promising at least , Precision and the five cards but ♦3 is an artificial request standard system in many countries FIVE OR SEVEN DEAL for opener to bid 3♥. With long hearts in continental Europe use five-card If the likely outcomes are that declarer and a weak hand, responder then majors. makes either eleven tricks (a five-level passes; with long spades, he converts to Responder can happily give a single contract) or thirteen, this is a five or 3♠. A development of the convention raise from 1♥ to 2♥ or 1♠ to 2♠ with seven deal. allows the partnership to play in 4♣ three-card support. Opinions vary or 4♦ (responder converts 3♥ to 4♣ about the wisdom of making a jump or 4♦). Nowadays, transfer bids have raise with the necessary values and ♠ K 10 5 rendered Flint almost obsolete. three-card support for partner’s major. ♥ A Q 9 6 5 2 When playing five-card majors, you ♦ 8 5 FLITCH often have to open ♣1 or 1♦ with fewer ♣ 7 2 A competition for married couples. than four cards in the suit. Responder generally strains to show a four-card N FLOGGER ♣ ♦ W E major in response to 1 or 1 as S A sheet recording the results of otherwise the partnership might miss previous rubbers. a 4-4 major-suit fit. ♠ A Q J 9 7 6 ♥ J 10 4 FORCE FIVE-CARD SPADES ♦ A 10 1 To make a forcing bid. For example: Some people play that you have to have ♣ A J 1♥-Pass-3♦. a five-card suit to open ♠1 but you can 2 To make an opponent ruff in order open 1♥ with only four hearts. With to shorten trumps in one of their such a method, the only hand shape Assuming the opening lead is a minor, hands – it is a common defensive that may require a prepared opening a contract in one of the majors is likely manoeuvre. in a minor is 4-3-3-3, where the four- to yield thirteen tricks if the heart card suit is spades. finesse works but only eleven if it FORCED BID fails – in the latter case, the defenders A bid that a player has an obligation FIVE-CARD STAYMAN score the winner they set up at trick to make, usually because the system After a 1NT or 2NT opening bid, ♣2 one as well as the ♥K. A grand slam is played so requires. Forced bids are or 3♣ asks partner to bid a five-card the best contract on this type of deal, very rare in natural systems like Acol major. The diamond response (2♦ or though this can be hard to diagnose in but do exist in relay systems. 3♦) denies the holding. If responder the auction. Forced passes are far more common. has a four-card major, he bids it on the Suppose you open 1NT and partner second round and opener will confirm FIX or FIXED raises to 3NT. You are always going a 4-4 major suit fit if one exists. A colloquialism meaning that a pair to pass because you have already In the UK, while it is quite common has received a bad score through no described your hand and your partner to play five-card Stayman in response fault of its own. For example, if your has chosen the contract. to 2NT, few pairs use it in response to opponents bid a slam with a trump 1NT. suit of A-8-7-6 facing J-5-3-2, it would FORCING DEFENCE Here is an example of the convention be a fix for you if it makes because one A strategy whereby the defenders keep in action: of you holds K-Q doubleton. playing cards that declarer must u

BRIDGE August 2020 Page 23 t ruff until declarer runs out of play undoubled. Both West’s pass West’s 2♠ creates a game force, trumps and the defenders gain control. over 3♠ and East’s pass over 4♠ are allowing East to make a simple raise forcing. setting spades as trumps. If East had 2 Systems utilising the opening call to jump to 4♠ over 2♠, it would be ♠ 9 8 6 2 of ‘Pass’ as a positive bid. Such hard for West, looking at three fast ♥ K 9 6 systems are legal only in very few heart losers, to explore for slam. ♦ 10 5 2 competitions and you are unlikely ♣ Q J 6 ever to encounter them. FORCING TWO BID ♠ Q J 10 3 ♠ A K 7 5 4 The use of an opening two-level suit ♥ 10 8 2 N ♥ J 7 4 3 FORCING SEQUENCE bid as an unconditional force. Playing ♦ A 6 4 3 W E ♦ Void A bidding sequence, which by it as a game force was a feature of the S ♣ K 4 ♣ 8 7 5 2 partnership agreement, is forcing. Culbertson and Goren systems. In ♠ Void Acol, it is usual to play a ♥ A Q 5 West North East South as forcing only for one round. ♦ K Q J 9 8 7 1♦ Pass 2♣ Pass ♣ A 10 9 3 2♠ FORFEIT Cancellation of rights, as appropriate West North East South under the Laws, after a misdemeanour. South plays in 5♦. West leads a spade 1♦ Dbl Pass at trick one and continues spades 2♦ FORWARD GOING after getting in with the ♦A and ♣K. A description of an encouraging bid. Having started with two trumps more It is usual to play the first sequence than West, ruffing three times means as forcing to game and the second FOULED BOARD that declarer ends up with one trump sequence as forcing to suit agreement. A board in which one or more cards fewer and the contract fails. The partnership should keep open the have gone back into the wrong place, bidding as long as necessary. meaning the next table cannot play FORCING 1NT the board. A convention whereby a response of FORCING TO GAME 1NT to an opening of one of a major is A forcing to game bid is a bid indicating FOUR-CARD MAJORS forcing for one round. It is usual to use that both members of the partnership Systems that do not guarantee more this in conjunction with the Precision should keep bidding until they reach than four cards in the suit when you system or with game forcing two-over- game. open one of a major suit. Acol and Blue one responses. After the forcing 1NT Once a game-forcing situation is Club are examples of systems utilising response, opener has to find a rebid in place, the partners can take the four-card majors. even with a 5332 hand type, bidding a bidding slowly to explore for the best three-card minor as necessary. contract, knowing that the other will FOUR DEAL BRIDGE not drop the bidding short of game. This is another name for Chicago, FORCING BID Examples of bids that create a forcing whereby a ‘rubber’ consists of A bid that requires partner to make to game situation in Acol are: a jump exactly four deals with pre-assigned at least one further bid. Forcing bids shift by opener or responder, opener’s vulnerabilities. include a change of suit response to an reverse after a two-level response, In most rubber bridge playing opening bid, an Acol 2♣ opening and at the three level, a clubs, four-deal bridge has replaced a 4NT ace enquiry. 2♣ opening with a suit rebid. traditional rubber bridge. This benefits Here is an example of a forcing to players waiting to cut in because game bid in action: rubbers finish more often. It also 1 A forcing pass forces partner to means that if you cut an unfavoured bid on or to double the opponents. partner, you do not have to wait long Often, when a player has to make ♠ K Q 9 4 ♠ A J 8 3 for a change. the decision between doubling a ♥ 7 4 3 N ♥ A 2 W E sacrifice bid made by an opponent ♦ A K Q 7 3 S ♦ J 2 FOUR SUIT TRANSFERS and bidding on, he may pass and let ♣ K ♣ Q 10 7 3 2 A method of showing length in any his partner make the final decision. suit by means of a transfer used in West North East South conjunction with simple red suit 1♥ Pass 3♦ 3♠ West East (Jacoby) transfers. After 1NT, Pass 4♠ Pass 1♦ 2♣ 2♠ 3♠ 2♦ = hearts After East’s strong response, it 4♣ 4♥ 2♥ = spades would be unthinkable to stop short 4NT 5♥ 2♠ = clubs of game or allow the opponents to 6♠ 2NT = diamonds

Page 24 BRIDGE August 2020 In the cases when responder shows a honour sequence. From Q-10-6-5-3, is very similar to a splinter, except that minor, opener has to choose whether you lead the five; from K-9-7-6-3-2, you are not bidding your shortest suit. to complete the transfer by bidding you lead the six; from J-8-4-2, you lead Playing fragment bids, 1♠-2♦-2♠-4♣ responder’s suit or to bid the suit/ the two. By leading low, you keep your suggests that responder has a main suit denomination below that. It is usual honours to capture those that declarer of diamonds, spade support and three to play one action as showing support holds and as entries. By leading or four clubs, hence a heart shortage. for responder’s suit and the other specifically fourth highest, you help as denying it. As is the case with red partner gauge your holding in the suit. FREAK suit transfers, responder may have a A hand or complete deal with an weak hand (intending to pass opener’s FOURTH IN HAND extremely abnormal distribution. The transfer completion) or a strong hand. The player who is fourth to call, i.e. the traditional definition of a freak hand player to the dealer’s right. for an individual player is one with a void or two singletons. Hand 1 Hand 2 FOURTH-SUIT FORCING ♠ 9 4 ♠ 9 4 After a partnership has bid three suits, FREE BID ♥ 5 ♥ A J 7 4 it is unlikely that the fourth suit is the A ‘free bid’ is one that you make ♦ J 10 6 2 ♦ A best fit. Therefore, you can usefully play after your right hand opponent has ♣ Q 9 8 7 4 2 ♣ A Q J 10 4 2 a bid of the fourth suit as an artificial overcalled your partner’s bid. The force, asking partner to describe his overcall ensures that your partner hand further. The bid does not promise will have another chance to bid On both hands, responder starts with any particular holding in the bid suit. and therefore you have the option 2♠, showing clubs. Traditionally, fourth-suit forcing is of passing. A ‘free bid’ in a suit will On the first hand, responder intends forcing for one round only, though it normally show a better than minimum to play in 3♣. is becoming quite common to play it as suit or extra values. On the second hand, the rebid will forcing to game. be 3♥. West North East South If you play four-suit transfers, you 1♥ Pass 2♣ 2♦ cannot bid 2NT to show a balanced ♠ K J 9 4 3 2♥ hand wishing to invite game. The ♥ 9 5 3 popular solution is to play non- ♦ K 5 Since West could pass over 2♦ knowing promissory Stayman, starting with 2♣ ♣ A Q 9 that East will have another call, 2♥ on such hands. shows either a six-card heart suit or An alternative solution is to play that perhaps a very good five-card suit eg a 2♦ response is either showing hearts West North East South with 100 for honours. or a balanced hand; with the balanced 1♦ Pass hand, responder has to make a specific 1♠ Pass 2♣ Pass FREE DOUBLE rebid, normally 2♠. ? At rubber bridge, it refers to the double of a game bid (or a partscore contract FOUR NO-TRUMP CONVENTION No natural bid describes your hand: which, if successful, would make Since 3NT suffices for game, one rarely 3NT is very risky without a stopper game whether doubled, undoubled or wants to play in 4NT. 4NT is usually a in the unbid suit; both 3♦ and 3♣ redoubled). conventional move towards a slam. The are underbids as well as overstating It is not of course ‘free’ but is likely most common uses are variations of your support for partner; 3♠ is also an to be less costly than doubling the the ace-asking convention, Blackwood. underbid and overstates your spades. opposition into game. The fourth suit bid of 2♥ is perfect. FOURTEEN THIRTY FREE FINESSE (1430) RESPONSES FOXU Term used to describe a finesse you Some users of Roman Keycard A rare conventional defence to an can take as declarer without losing the Blackwood play, either always or in opening three-level pre-emptive bid. trick should it fail. specific situations, that a 5♣ reply Fishbein applies sitting Over the For example, in the first layout shows 1 or 4 key cards while 5♦ shows bidder and double (X) sitting Under, below, when West leads low, you can 0 or 3. The idea is that the 4NT bidder as take out requests. finesse dummy’s jack. Should East is more likely to wish to investigate produce the queen, you can capture it further facing 1 key card than none. with the ace. An unusual jump or double jump In the second layout, assuming this FOURTH-HIGHEST LEAD bid showing a fit for partner’s suit, a is a side suit, you can finesse the queen The lead of the fourth highest card in few cards in the suit bid and shortage when West leads it, knowing that you a suit. This lead is standard unless the elsewhere. can ruff if East produces the king. suit contains no honour or contains an In terms of concept, a fragment bid u

BRIDGE August 2020 Page 25 t yield enough tricks to make game. ♠ 9 6 3 K J 2 A Q 5 ♥ 8 6 3 GAME FORCING BID ♦ K Q 8 2 A bid that demands that the N N W E W E ♣ 9 6 2 partnership does not stop short of S S ♠ A Q 8 5 ♠ J 10 7 game. In Acol, common examples are: A 3 Void ♥ 9 4 2 N ♥ Void a jump shift by responder (eg ♥1 -Pass- W E ♦ J 10 S ♦ 9 7 6 5 4 3 2♠) or opener (eg 1♥-Pass-1♠-Pass-3♣) ♣ K Q J 5 ♣ 10 7 4 3 or opener’s reverse after a two-level FRIGID ♠ K 4 2 response (eg 1♦-Pass-2♣-Pass-2♠). Slang term for a contract that is certain ♥ A K Q J 10 7 5 to make. ‘Cold’ and ‘Icy’ are similar ♦ A GAME FORCING SITUATION terms. ♣ A 8 A sequence of bidding that has committed both members of a FROZEN SUIT partnership to reach a game contract. A suit that is disadvantageous to lead. South plays in 4♥ after West opened The situation arises whenever one or Frozen suits commonly occur when the bidding. West leads the ♣K, which the other member of the partnership each of the four players has some you allow to win. You win the second has made a game forcing bid. strength in the suit. club and cash a top trump, finding East The big advantage of a game forcing If West or East leads this suit, North- void. You should continue by cashing situation is that the partners can South make three tricks. If North or the ♦A and leading a low heart. You take the bidding slowly to describe South leads the suit, East-West make a are giving up a trump trick but, by their hands and explore for the best third-round winner. securing an entry to dummy with the contract knowing that the bidding ♥8, you get tricks with the king and will not suddenly stop. queen of diamonds in return. A 10 5 GAME HOG

N GAMBLING 3NT A player who habitually distorts his Q 7 4 W E J 8 6 3 An opening bid of 3NT to show a long, own bidding to increase his chance of S solid minor, at least A-K-Q-x-x-x-x, being declarer in game contracts. K 9 2 with no more than a queen outside. Responder usually passes (with GAME IN stoppers in the other suits) or retreats Another expression for ‘Vulnerable’. FRUIT MACHINE SWISS to 4♣, asking opener to pass (with A convention for replying to partner’s clubs) or correct to 4♦. GAME INVITATION 1♥ or 1♠ opening that has fallen into or GAME TRY disuse. It uses both 4♣ and 4♦ as GAME A bid that does not force the conventional raises of opener’s major, The 100 points scored below the line partnership to game but invites both with four trumps, game values in rubber bridge. 100 or more trick partner to bid it with extra values, in and two aces. The 4♣ bid shows an points scored on one deal in duplicate the context of his previous bidding. extra feature: a third ace, a singleton or bridge. For example if partner opens 1NT the king of trumps. The 4♦ bid denies and you raise to 2NT, this is a game those extra features. GAME ALL invitation: you expect partner to bid In rubber bridge when both sides 3NT if maximum. have previously won a game and are thus both vulnerable. In duplicate or GAME VALUES Chicago when the pre-determined The high cards sufficient to provide vulnerability so indicates. an expectation of making game. Most authorities regard the minimum G GAME BID values as 25 HCP for 3NT, 26 points A game bid is a bid at just high enough for 4♥ or 4♠ and about 28 points for a level for game: 3NT, four of a major 5♣ or 5♦. or five of a minor. In rubber bridge, GADGET if the partnership already has points GAME TRY DOUBLE A convention or part of a convention. towards game, the bid may be at a level A double made in a competitive sufficient to convert that partscore auction, when both sides have bid and GAMBIT into game. supported a suit; this is a means of The deliberate sacrifice of a trick in distinguishing between a competitive order to gain two or more other tricks. GAME CONTRACT raise of partner’s suit and a game Any contract that if successful will invitation. The double is the game

Page 26 BRIDGE August 2020 invitation, and the immediate raise is unseen hands either as declarer or Over 1♣: thus purely competitive. defender. 2NT Shows the red suits You and your partner bid all the 2♦ Shows the majors West North East South way to 7♠ (or you may be playing 3♣ Shows the other suits 1♥ 2♦ 2♥ 3♦ matchpoint pairs and be keen to make (diamonds and spades) Dbl an overtrick in 6♠). West leads a trump and East follows. The modification after an opening♣ 1 Double here is West’s only way to try You play a second round of trumps bid came about to allow an overcall of for game. and all follow. 2♣ to be natural because, in France, To maximise your chance of working a 1♣ opening is often on a three-card GARBAGE out who has the ♣Q you want to find suit. Another modification of the Slang term for a poor hand. out how many clubs each defender convention uses the cue bid has – whoever has more clubs is the after a minor suit opening to show the GARDENER 1NT OVERCALL favourite to hold the queen. extreme suits (spades and the other A rare convention by which an minor) and the jump cue bid (eg 1♣- overcall of 1NT shows either a strong 3♣) to show the majors. no-trump (15-17 or 16-18 points) or a ♠ A Q 10 7 When both members of the weak hand with a long suit. ♥ A 8 6 2 partnership remember which suits the ♦ A 10 Ghestem bid shows, the convention GENERAL LICENCE ♣ K 10 6 works well. However, Ghestem seems A licensing category of the EBU laws to spark more memory lapses than and Ethics Committee relating to N almost any other convention. W E treatments and conventions that they S have approved for most competitions. GIN ♠ K J 9 4 2 Slang term meaning that a contract is GENERAL PURPOSE CUE BID ♥ 4 certain to succeed. An uncommon treatment whereby a ♦ K 8 4 2 bid of 4NT serves as a general try for ♣ A J 8 GIVE COUNT a slam when a cue bid is not available To make a distributional signal. For or convenient. example, if as a defender you hold You cash the red suit tops, ending in Q-9-6-3-2 in a suit, playing the two GERBER dummy and ruff a heart. You then would be a count signal, showing an A convention whereby a bid of 4♣ crossruff the red suits, noting what odd number in the suit. asks about the number of aces held by happens. partner. In response: Suppose all follow to four rounds GLADIATOR of hearts but East shows out on the A system of responses to a 1NT 4♦ Shows zero or four aces fourth round of diamonds. In this opener. 2♣ shows a weak take-out 4♥ Shows one ace case, West’s shape is 2-4-4-3 and East's type of hand with length in any suit. It 4♠ Shows two aces is 2-4-3-4, so you play East for the ♣Q. demands 2♦, which responder can pass 4NT Shows three aces or convert to 2♥ or 2♠. 2♦ over 1NT is GET THE HAND the Stayman enquiry (2NT denying a Subsequently, 5♣ enquires about kings OFF YOUR CHEST four-card major), while 2♥ and 2♠ are with corresponding responses. A colloquial expression to mean forcing with a five-card suit. Due to the usefulness of a 4♣ bid making a bid that fully expresses your A jump to three of a major is a slam for other meanings, such as a cue hand early in the auction. invitation. bid, a splinter or as a natural bid, it is common to play that 4♣ is only GHESTEM GO DOWN Gerber in very limited situations, such A system of usually strong two-suited To make fewer tricks than contracted as a direct response to a 1NT or 2NT devised by Pierre Ghestem of for, also to quantify the extent of doing opening. Gerber is one of the most France: so. For example: If you are in 4♠ and abused conventions ever invented. make nine tricks, you go one down. Over 1♦ /1♥/1♠: GESTURE 2NT Shows the lowest GO IN or GO UP A remark or mannerism that suggests two unbid suits A defender in second position who satisfaction, dissatisfaction or a call, 3♣ Shows the highest plays the ace when declarer leads low lead or play. two unbid suits towards dummy ‘goes in’ or ‘goes up’ Cue bid Shows the other with the ace. GET A COUNT OF THE HAND two suits To discover the distribution of the u

BRIDGE August 2020 Page 27 t unions; Premier Grand Master is now If North has several winners in some GO OFF the highest rank. other suit but no obvious means of To make fewer tricks than contracted entry, South might cash the ace and for. GRAND SLAM then lead the nine. If East accepts the To bid a Grand Slam is to contract to gift and wins with the ten, the eight GO TO GAME make all thirteen tricks. becomes an entry to all those winners. To bid a game. GREEN A convention invented by Ely and 1 This is one of the traffic-light The premier Knock-Out Teams Josephine Culbertson but was first terms that the Laws and Ethics Championship of Great Britain. It is published in the US by Josephine in Committee of the English Bridge contested under the auspices of Bridge and hence often Union uses to categorise psychic Great Britain. referred to as ‘Josephine’, whereby bids. A ‘green’ psyche is one where a direct bid of 5NT (not preceded the partnership’s subsequent GOOD by 4NT) after a trump suit has been actions provide no evidence of 1. In hand valuation, a way to describe agreed asks partner how many of the an unauthorised partnership that a particular feature or the hand three top trumps honours (ace, king understanding. as a whole is better than average. and queen) he holds. 2 Shorthand for describing the For example, a hand with 13 HCP, a Originally, responder bid seven of vulnerability of both partnerships 5-card suit, and a couple of tens you the trump suit with two or three top on a board and meaning that the might say is a good 13 points. honours and six of the trump suit with ‘green’ partnership is not vulnerable 2. In the play, used to describe a hand none or one. Later methods give better but their opponents are vulnerable. in which every card is a winner, definition, for example if spades are eg ‘Dummy is good’ or a card that trumps: GREEN POINTS is a winner. ‘High’ has the same National Master Points issued by the meaning in this sense. 6♣ No top honour English Bridge Union (for success in 6♦ One top honour certain larger tournaments). Their GOREN, Charles H 6♥ Two top honours accumulation is necessary to attain the American bridge player, successful 6♠ Three top honours higher grades in its ranking system. in converting the bridge playing world from honour tricks to point Here is a hand on which you might use GROSVENOR COUP count valuation and founder of much the Grand Slam Force: An intentional misplay that allows modern American bidding theory. an opponent to gain a trick but is so ‘stupid’ that the opponent will never ♠ Q 7 5 believe the misplay has occurred. For A method of dealing when you do not ♥ Void example: shuffle after the previous deal’s play. ♦ A K Q J 9 7 The dealer starts by giving five cards ♣ A Q 6 4 at once to each player. This is repeated ♠ A J 4 for a second round, before a further N three cards are dealt together to each If partner opens 3♠ (or shows a long ♠ 3 W E ♠ Q 10 2 player. spade suit by some other sequence), S An alternative method is to deal you want to reach 7♠ unless a top ♠ K 9 8 7 6 5 four cards to each player on the first trump is missing. You find out by round and then three cards to each bidding 5NT. player on three subsequent rounds. As North cashes the ♠A and then leads one might expect, hands dealt this way GREEK GIFT the ♠J. East plays a ‘Grosvenor Coup’ are often wildly distributional. A trick offered to the opposition that, by underplaying the two then the ten. if accepted, leads to disaster. For South will never believe that East has GRAND COUP example: failed to play the queen on the jack and A in which winners must so will play his king hoping for a 2-2 be ruffed to shorten declarer’s trumps. break. The name comes from the early days of 8 3 2 whist when players thought that such a N play deserved a high-sounding title. Void W E 10 6 4 A rare triple squeeze in which one S opponent holds stoppers in two suits GRAND MASTER A K Q J 9 7 5 with a holding shorter than a guard in For a long time this was the highest a third suit that protects his partner Master Point rank of the home bridge from a finesse, for example:

Page 28 BRIDGE August 2020 HALF STOPPER less help from partner to make 3NT ♠ K 4 A holding that is not quite good enough than 5♣ or 4♥. In any case, North is ♥ A J 6 to prevent the run of an opposing suit, most unlikely to lead the ♠A. Bid 3NT. ♦ — such as Q-x or J-x-x. ♣ — If both members of the partnership HAND ♠ J 7 ♠ Q 9 3 have a half stopper in the same suit, 1. The thirteen cards held by a player; ♥ K Q N ♥ 4 they will have a stopper between them, 2. The set of four hands (now more ♦ A W E ♦ — S hence the name half stopper. commonly referred to as the deal); ♣ — ♣ 5 3. The position the player occupies at ♠ A 10 6 HALF TABLE the table, as in ‘first in hand’ (the ♥ — A table at which only one pair is dealer). ♦ K sitting. ♣ A HAND PATTERN HALF TRICK The way in which the cards, when An honour holding that one can arranged into suits, are within one South leads the ♣A, squeezing West in expected to win a trick half the time hand. For example: 5-4-3-1 ‘pattern’. three suits. He has stoppers in hearts such as K-x, or the queen in a holding This is also the hand’s ‘distribution’ or and diamonds and a partial guard of A-Q (one and a half tricks). ‘shape’. in spades. To avoid giving away a The hand for the Hamman’s rule certain trick, he discards a spade. This HALMIC example has a 6-3-3-1 pattern. enables South to play spades and take An escape mechanism after an the finesse against East on the second opponent doubles your partner’s 1NT HANDICAPPING round. opening. To impose a penalty on the stronger You redouble to show an unspecified members of the field to allow the GUIDE CARD 5-card suit; opener must bid 2♣. You weaker players a chance of victory. A card used to instruct contestants to then show your long suit, passing 2♣ An alternative method of which table they should move after the if it is clubs. Immediate bids after handicapping is to restrict the number end of each round, either given to each 1NT doubled show two-suited hands of players over a certain masterpoint contestant, or placed on each table. (possibly with only 4-4 in the suits) rank that can play in a team or in a with the suit bid and a higher-ranking partnership. GUARD suit. 1. This is a card combination that After the two suited-bids, opener HARD VALUES prevents the opposition from passes with some fit for the suit bid Aces and kings. By comparison, running a suit – the same as a and otherwise bids the next suit up to queens and jacks are ‘soft values’. stopper but generally used when find out responder’s second suit. referring to a defender. HELD 2. A small card held with an honour to HAMMAN’S RULE An abbreviation for a popular discard ensure that the honour does not fall This says that if you have a choice of method, High Encouraging Low under higher honours eg with K-2, bids and one of the options is 3NT, Discouraging. Similar terms are the two is a guard to the king: if bid 3NT. The idea is that 3NT requires ‘natural discards’ and ‘rubber bridge one had to discard the two, the king the fewest tricks for game and, even discards’. would become unguarded. if there is a winning defence, the Here is an example: defenders may get off to the wrong lead. ♠ Q ♥ K Q J 7 4 2 ♠ K ♦ A 9 2 ♥ 9 4 3 N ♣ 9 6 3 H W E ♦ A 7 4 S ♣ A K Q J 8 3 Suppose you lead the ♥K, losing to the HACKETT ♥A. On the second round of spades, A now rarely played defence to weak West North East South you can discard a high diamond, the two openings whereby 3♣ is a take-out 3♠ Pass Pass nine, to encourage diamonds, or a low request showing less than 16 points; ? club, the three, to discourage clubs. 3♦ is a take-out request showing 16+ Either discard should persuade points. The options here are 3NT, 4♣ and partner to look for an entry in your double. Although you do not have a hand in diamonds rather than clubs. certain spade stopper, you need far u

BRIDGE August 2020 Page 29 t K-Q-J-4 facing A-5-3-2, the five could HINDSIGHT HELP SUIT GAME TRY be a hidden entry – when crossing to Frequently expressed by players in This type of game try applies after the ace, declarer leads an honour and their post mortem of hands that they you open one of a major and partner overtakes, setting up the five as a fourth- might have bid or played better. makes a single raise. A bid of a new round entry on the normal 3-2 break. suit asks partner to bid game with help HIT in the suit bid. HIGH CARD Colloquialism for double. 1. An ace, king, queen or jack is a high card – the same as a court card. HLQT ♠ A Q 9 6 4 ♠ K 10 3 2 2. In the context of defensive signals, A method for determining whether ♥ A K N ♥ 9 6 5 it usually means a high spot card. a hand is worth an opening bid. It W E ♦ Q 8 6 3 S ♦ K 7 3. A card that has become a winner is stands for High cards Length Quick ♣ 8 4 ♣ 10 6 3 2 ‘high’. Tricks. You add your high card points to the lengths of your two longest suits HIGH CARD POINTS (HCP) and your number of quick tricks. If the West East Numerical measure of a hand’s playing total comes to 22 or more, you should 1♠ 2♠ and defensive strength. An ace counts open even if vulnerable. 3♦ 4♠ as four points, a king as three points, a queen as two points and a jack as one. West is not quite good enough to bid Distributional assets, such as a long ♠ A 10 6 4 2 4♠ over 2♠ and so makes a help suit suit or a void do not count. There are ♥ K J 7 4 game try. East has an ideal diamond 40 HCP in a pack. ♦ K 2 holding as well as good trumps, so ♣ 5 3 accepts the game invitation despite HIGH-LOW SIGNAL having only 6 HCP. The play by a defender of an unnecessarily high card on the first You have 11 HCP, 9 cards in the majors HERBERT NEGATIVE round of a suit followed by a lower and 2 quick tricks (the ace is one and A bid of the next step up (eg 2♥- card on the second round to signal to the kings are a half each). Open 1♠. Pass-2♠) as a negative response to partner. Usually the peter shows that a strong two opening bid. Over 2♠, the signaller likes the suit or that he HOG some players use 2NT as the negative has an even number of cards in the A person who attempts to become the response, others 3♣. suit. ‘Peter’ and, especially in America, declarer as often as possible. Thus the ‘Echo’ are synonymous terms for phrase ‘to hog the bidding’. HESITATION this signal. For example, if, in a suit The proprieties of bridge dictate that contract your partner cashes an ace- players should bid and play in even king, you would usually play high-low Decline to win a trick, often with the tempo and rhythm. An unusually long with a doubleton to signify that you are intention of disrupting opponents’ pause by a player before bidding or happy to have a third round led to ruff. communications. This is a common playing to a trick would be an undue technique and is available to declarer hesitation. The player’s partner then HIGH REVERSE and the defenders. has an obligation under the Laws to A rebid by opener in a lower ranking ignore any unauthorised information new suit at the three level after a two- gained from such a hesitation. over-one response, eg 1♠-Pass-2♥- ♠ Q 4 You should take particular care Pass-3♦. It is normal to play this bid as ♥ 8 6 5 3 not to hesitate and then pass as this forcing to game. You need at least five ♦ K Q 9 5 2 conveys the information that you were cards in your first suit, at least four ♣ 9 2 thinking of taking a stronger action cards in your second and better than ♠ J 10 9 6 3 ♠ A 7 5 than passing. You should also avoid a minimum opening. Your second suit ♥ Q 9 4 N ♥ J 10 2 ♦ W E ♦ hesitating if you hold a singleton in may be shorter than your first suit or the 8 6 S A 7 4 the suit led or have some other forced same length. For the sequence quoted, ♣ 10 6 3 ♣ J 7 5 4 play because it is against the rules to either of these hands would be fine: ♠ K 8 2 mislead your opposition by doing so. ♥ A K 7 ♦ J 10 3 HIDDEN ENTRY Hand 1 Hand 2 ♣ A K Q 8 A low card, usually in the dummy, that ♠ A Q 10 7 4 ♠ A J 9 7 5 3 can become an entry with careful play. ♥ 3 ♥ 3 Usually declarer plays one or more of ♦ A K J 8 3 ♦ A K Q 4 South plays in 3NT and West leads the his high cards of a suit under the high ♣ J 2 ♣ K 6 ♠J. Declarer calls for the queen, which cards in dummy. For example, with East wins with the ace. Back comes a

Page 30 BRIDGE August 2020 spade. the total number of honour tricks in competitive play with as few as three If declarer wins the second spade, each suit: tables. East will have a third spade to lead Unlike Mitchell movements, pairs upon getting in with the ♦A. So A-K Two honour tricks change directions at some tables. In declarer's correct play is to hold up the A-Q One and a half a full Howell movement, all the pairs ♠K until the third round. honour tricks will play all the other pairs. Declarer plays on diamonds and it A-K-Q Three honour tricks In terms of fairness and sociability, is East’s turn to hold up. Winning the K-J-10 One honour trick a Howell scores highly. However, the first or second diamond would allow K-x, Q-J-x Half an honour trick shortage of stationary pairs and the the diamond suit to produce four lack of flexibility about the number tricks. Instead, East holds up the ♦A In addition ‘plus values’ were deemed of boards played means that their twice, taking it on the third round. to be worth approximately one use is relatively infrequent. Howell quarter of an honour trick. These movements for 5 tables (9 3-board HOLD plus values were: any queen that was rounds) and 7 tables (13 2-board 1. To hold a card means to possess it not a singleton, any jack supported by rounds) are the most popular. in your hand. another honour (but not a doubleton 2. If you lead a card and it wins the combination nor in a suit holding of A HUDDLE trick then it ‘holds’. -K-Q-J), any singleton or void (but not A prolonged hesitation. more than one). HOLDING High Card Points are an older HUM Particular cards in a player’s hand, as valuation method than Honour Tricks With the advent of many weird and in ‘a good club holding’. and after Culbertson’s influence on wonderful systems and conventions, the bridge world waned, few players the Laws have deemed some as: HOME BRIDGE UNION continued to use Honour Tricks as ‘Highly Unusual Methods’ or HUMS. In the UK, this is any of the English their main valuation method. The vast majority of competitions and Bridge Union, the Scottish Bridge clubs prohibit the use of Hums. Union, the Welsh Bridge Union and HONOURS the Northern Ireland Bridge Union. In rubber bridge or Chicago any player HYBRID SCORING holding four or five honour cards in A form of scoring that combines the HONEYMOON BRIDGE the trump suit or holding all four aces best features of aggregate or IMP A term to describe various forms of in a no-trump contract may claim for scoring and point-a-board scoring. two-handed bridge. ‘Honours’ and score a bonus. Used in top-level contests, where two teams play only short matches, such as HONOUR HOOK the EBU’s inter-county Pachabo Cup. One of the five highest cards in a suit, Colloquial term for taking a finesse. ie the ace, king, queen, jack or ten. HOST HONOUR LEAD A volunteer at a duplicate club who is The lead of an honour card. Unless available to play if someone turns up it is from a short suit, the lead of an without a partner. I honour in standard methods promises the honour immediately below it. HOUSE PLAYER For example, the lead of the jack A player at a rubber bridge club would be consistent with holdings ‘employed’ by the management to ICY of J-10-9-x, J-10-8-x-x, K-J-10-x-x make up tables. Slang term for a contract that is certain or J-10-x. Against a suit contract, to make. ‘Cold’ and ‘Frigid’ are similar you would also lead the jack from HOUSE RULES terms. J-10-x-x. Against a no-trump contract, Additions or amendments to the you would also lead the jack from Laws that a club or group may require IDLE CARD A-J-10-x-x. players to meet. Such rules would A card not required for a specific apply mainly to matters of dress, purpose as distinct from a busy HONOUR STRENGTH periods of play, stake limits etc. A club card. The term usually applies The value of a hand in terms of honour may also adopt different rules from to unimportant cards in squeeze cards. its National Bridge Organisations on positions. which conventions are permitted. HONOUR TRICKS ILLEGAL CALL As defined by Culbertson, an honour HOWELL A call in the course of the bidding trick was the basic unit of defensive Cyclic pairs movement, usually auction that is insufficient, out of value. The overall value of a hand was dictated by movement cards, allowing rotation, or otherwise improper. u

BRIDGE August 2020 Page 31 t of play is lower than in normal predetermined chart. Computer scoring For example: events because inevitably the casual has made the use of instant matchpoints partnerships have misunderstandings. very rare in duplicate events. West North East South 1♥ Pass 2♦ Pass INFERENCE INSUFFICIENT BID A conclusion drawn about the likely A bid not legally sufficient, below the By West, a double, redouble or bid of lie of the cards from the previous play level of the minimum allowable bid eg or below 2♦ would be illegal. Any call and bidding. Inferences are a very 2♣ after someone has bid ♦2 . The Laws by one of the other players would be important part of the game. Suppose apply. illegal. declarer can see this spade suit: INSULT IMP The bonus of 50 (100) points awarded This is an abbreviation for ♠ 9 6 4 for bidding and making a doubled

International Match Points (IMPs), N (redoubled) contract is commonly the standard method for scoring W E referred to as ‘50 (100) for the insult’. S teams-of-four matches. ♠ J 3 INSURANCE BID IMPOSSIBLE NEGATIVE Sacrifice bid against a high-level In the system, after the contract by the opposition despite strong 1♣ opening, a positive response West bid spades, East did not support some expectation of defeating that in a suit shows a five-card suit while and West leads a low diamond against contract. Insurance bids are more 1NT shows a balanced hand. To show South’s heart contract. The first common at aggregate (rubber) scoring 4-4-4-1 hands with positive values, inference is that spades are probably and IMPs than at matchpoints. In the one starts by giving the negative 6-2: if they were 5-3, East might have former, you do not want to concede a response of 1♦ and later one makes a supported. The second inference is large score when you might have made bid inconsistent with holding negative that East has the king of spades; if East a score yourself or conceded a small values, thus indicating an ‘impossible did not hold the king, West would hold penalty. At matchpoints, whether you negative’. a sequence of honours in the suit and are plus or minus assumes greater would very likely have led one. You importance. IMPROPRIETY can also infer that West does not hold A breach of ethical conduct. an honour sequence in diamonds. ♠ J 9 5 3 IN FRONT OF INFORMATORY DOUBLE ♥ 7 Term describing the position of a Old term for a take-out double. The ♦ K 9 8 5 3 2 player with respect to his left-hand term arose as the double ‘informed’ ♣ 9 4 opponent and therefore having to play partner that the doubler held opening before him. values. West North East South INCOMPLETE RUBBER INHIBITORY DOUBLE 4♥ 4♠ 6♥ If a game of rubber bridge has to finish A psychic manoeuvre to make declarer ? before one side has won two games, panic into thinking that suits are there are bonuses awarded depending breaking badly or that key are The haphazard nature of the auction on the state of the rubber. A rubber likely to fail. Frowned on nowadays means you cannot be sure 6♥ is might finish incomplete if it is still in because it succeeds only against making. However, you do know that play at a club’s closing time or one of inexperienced players. 6♠ doubled is going to be no more the players has to leave. than a couple off. At rubber or IMPs, INSPECTION OF TRICKS you would take out insurance by INCOMPLETE TABLE In rubber bridge, any player may bidding 6♠. A table at which fewer than the inspect the previous trick until his necessary four players are sitting, side has played to the next trick. In INTERFERENCE BID especially a half table at duplicate. duplicate, he may not do so after he Any defensive overcall made to has turned his own card over. He may obstruct the opponents’ bidding. It is INDIVIDUAL MOVEMENT inspect, but not expose, his own card not strength showing or constructive. A movement used in an event where until someone has led to the next trick. Jump bids may be constructive for pre- individual players score separately, emptive interference. For example: competing with different partners INSTANT MATCHPOINTS against one another. A method of scoring in a one-table West North East South Individual movements are great situation or a duplicate event by Pass Pass Pass 1♥ social occasions, though the standard comparing the score obtained with a 2♦

Page 32 BRIDGE August 2020 Since neither West nor East could INTERMEDIATE JUMP 1500 – 1740 17 open the bidding, it is unlikely that OVERCALLS 1750 – 1990 18 they will buy the contract. West’s A jump overcall based on an opening 2000 – 2240 19 interference overcall reduces North’s hand with a six card-suit. These are not 2250 – 2490 20 responding options and will help very popular as you can start with a 2500 – 2990 21 East with the opening lead if North simple overcall on such hands whereas 3000 – 3490 22 becomes declarer. it is less easy to find a good alternative 3500 – 3990 23 way to bid weak and strong hands. 4000 + 24 INTERIOR SEQUENCE A sequence of honour cards (including INTERMEDIATE TWO BIDS Suppose a team’s scores are +420 in the 9) that comprises two or more An opening bid of 2♥ or 2♠ to show one room and -170 in the other. The touching cards with one higher non- a six-card suit with about 13-16 points swing is 250, which translates to a gain touching honour. (less with a seven-card suit). Using of 6 IMPs. If a team scores -100 in one For example, A-Q-J, K-J-10, Q-10- such a method, opening one of a major room and -600 in the other, the adverse 9. The standard lead from an interior and rebidding two of the major implies swing is 700, which translates to a loss sequence is the highest of the touching a hand weaker than this; opening one of 12 IMPs. cards, ie the queen, jack and ten of a major and rebidding three of the respectively. major promises a hand stronger. INTERVENING BID Here is the interior sequence lead in An overcall. action: INTERNATIONAL BRIDGE PRESS ASSOCIATION (IBPA) INTRA FINESSE A worldwide organisation whose A finesse against a lower honour held J 7 3 members are either bridge authors or by one opponent in preparation for a newspaper columnists. finesse against a higher honour held by N Q 10 9 6 W E K 4 2 the other opponent. For example: S INTERNATIONAL MATCH A 8 5 POINTS (IMPs) A method of scoring used in teams ♠ Q 9 3 matches; it converts the aggregate N After West leads the ten and dummy difference between the scores of ♠ 10 2 W E ♠ K J 7 6 5 plays low, East can withhold the king, opposing teams on each board to S allowing the defenders to finesse International Match Points on a ♠ A 8 4 against dummy’s jack. Had West led defined sliding scale. The purpose is to a fourth best six, East would have to reward large swings more generously put up the king to stop the eight from than small ones but to limit the extent If, as South, you have reason to place scoring, setting up the jack as a third- of very large swings to prevent the East with the ♠K, you reject the round winner. result on one board from effectively normal play of leading up to the queen. determining the outcome of an entire Instead, you finesse the nine, losing to INTERMEDIATE CARDS match. the jack. When you then get the lead in Tens, nines and eights. The average the North hand, you lead the queen to hand has one ten, one eight and one Aggregate IMP pick up East’s king and West’s ten. nine. If you have significantly more Difference Difference or fewer intermediate cards than 0 – 10 0 INVERTED MINOR SUIT RAISES average, it is usual to adjust your hand 20 – 40 1 This convention ‘inverts’ the normal valuation accordingly. 50 – 80 2 meanings of the single and double 90 – 120 3 raises of a minor suit. Thus 1♣-Pass- 130 – 160 4 2♣ is stronger than 1♣-Pass-3♣. The Hand 1 Hand 2 170 – 210 5 single raise is forcing for one round, ♠ Q 7 5 ♠ Q 10 5 220 – 260 6 enabling responder to make the bid ♥ K 8 2 ♥ K 10 2 270 – 310 7 on both invitational and game-going ♦ A J 7 4 ♦ A J 9 4 320 – 360 8 hands. ♣ Q 7 2 ♣ Q 9 370 – 420 9 Playing inverted raises, you would 430 – 490 10 raise 1♣ to 3♣ or 1♦ to 3♦ with 500 – 590 11 Hand 1. Even if this contract does If partner shows a balanced 12-14, you 600 – 740 12 not make, it is very likely that the would merely invite game with Hand 750 – 890 13 opponents could have made 2♥ or 1 (no tens or nines) but bid 3NT with 900 – 1090 14 2♠. The benefit of playing inverted Hand 2 (two tens and two nines). 1100 – 1290 15 raises is even clearer if you hold the 1300 – 1490 16 second hand and partner opens 1♦. u

BRIDGE August 2020 Page 33 BERNARD t You can bid 2♦, forcing. In lead in hand and draw two rounds of standard methods, you would have no trumps with the ace and king. When MAGEE’S satisfactory response. you discover the 3-1 split, it would be risky to attempt a fourth-round TUTORIAL diamond ruff in dummy. Hand 1 Hand 2 Instead, you play for a squeeze. You CD-ROMs ♠ 7 5 ♠ 7 5 want to arrive at a position where only Compatible with Apple Mac ♥ 9 3 ♥ A Q 3 one defender is guarding both red OS X 10.8 - 10.14 ♦ K J 5 3 ♦ K J 10 5 3 suits. You cross to the ♥A, ruff a heart ♣ Q 9 7 5 2 ♣ K 6 4 high, back to the ♠Q and ruff another ACOL BIDDING heart high. l Opening Bids and After three rounds of hearts, you can Responses INVITATIONAL BID be certain that only one defender can l Slams and Strong Openings A bid that encourages partner guard the suit. When you cash your l Support for Partner to bid on when holding any last trump and cross to the ♣K, East l Pre-empting £66 extra values in the context of his has no answer. l Overcalls previous bidding. For example, l No-trump Openings 1♣-1♦-1NT-2NT invites partner to ITALIAN STYLE CUE BIDS and Responses ♥ ♠ ♥ ♥ l Opener’s and Responder’s bid 3NT while 1 -1 -2 -3 invites This style of cue bids, developed by the Rebids partner to bid 4♥. Italian , has now become the l Minors and Misfits norm amongst tournament players. l Doubles ISOLATING THE MENACE Below the level of 4NT, you treat first- l Competitive Auctions A method of leaving only one player and second- round controls equally. in the position of guarding a particular DECLARER PLAY suit, thus increasing the chance West North East South 1♥ Pass l Suit Establishment in of executing a successful squeeze. 1♠ Pass 3♠ Pass No-trumps Technically the more correct term is 4♦ l Suit Establishment isolating the guard. in Suits £76 l Hold-ups West’s 4♦ shows first- or second- round l Ruffing for Extra Tricks ♠ Q 3 2 control in diamonds (ace, void, king or l Entries in No-trumps ♥ A 10 8 5 2 singleton) and denies first- or second- l Delaying Drawing Trumps ♦ 6 5 4 round control in clubs. East, if lacking l Using the Lead ♣ K Q first- or second- round control of clubs l Trump Control ♠ 8 6 5 ♠ 7 knows immediately that a slam is not a l Endplays & Avoidance ♥ K J 4 N ♥ Q 9 6 3 good proposition. l Using the Bidding W E ♦ 9 8 ♦ J 10 7 2 S ♣ J 10 9 3 2 ♣ 8 7 5 4 ITALIAN STYLE TAKE-OUT DEFENCE ♠ A K J 10 9 4 DOUBLES ♥ l Lead vs No-trump 7 A style, widely regarded as unsound Contracts ♦ A K Q 3 but used to good effect by the Italian l Lead vs Suit Contracts ♣ A 6 Blue team, of making take-out doubles l Partner of Leader vs of suit openings on weak no-trump No-trump Contracts hands either when the doubler has a l Partner of Leader vs You and North do well to reach 7♠ 4-3-3-3 shape or when the doubler’s Suit Contracts played by South. You win the club doubleton is in an unbid suit. n l Count Signals l Attitude Signals l Discarding £76 l Defensive Plan DUPLICATE BRIDGE l Stopping Declarer l Counting the Hand RULES SIMPLIFIED by David Stevenson ALSO (otherwise known as the Yellow Book) only l Advanced Acol Bidding £595 l Advanced Declarer Play FULLY REVISED IN 2017 l 5 Card Majors See p44 ( 01483 489961 Available from Mr Bridge www.mrbridge.co.uk/shop ( 01483 489961 www.mrbridge.co.uk

Page 34 BRIDGE August 2020 Answers to Slam Bidding Part 1 Quiz on page 13

1 Your partner opens 2NT (20-22 Auction 2A Auction 2B Auction 2C Auction 3A Auction 3B Auction 3C points). Playing simple Acol what do West East West East West East West East West East West East you respond with Hands 1A, 1B and 1♥ 1♠ 1♥ 2♣ 1♥ 1NT 1NT 2♣ 2NT 3♣ 2NT 3♦ 1C? 2♥ 3♣ 2♥ 3♣ 2♥ 3♦ 2♥ 4♦ 3♥ 4♦ 3♥ 4♦

Auction 2D Auction 2E Auction 2F Auction 3A: 4♦ is a cue bid agreeing Hand 1A Hand 1B Hand 1C West East West East West East hearts. 4♦ shows first ♠ 9 6 3 2 ♠ A Q J 2 ♠ 9 6 3 2 1♥ 1NT 1♥ 1♠ 1♥ 2♦ round control in diamonds ♥ Q 5 4 ♥ Q 10 9 ♥ K Q 4 2♦ 3♣ 3♥ 4♦ and denies first round ♦ Q 6 5 ♦ 8 7 ♦ Q 6 5 control in clubs. ♣ A Q J ♣ Q 10 9 4 ♣ A Q J Auction 2G Auction 2H Auction 2J Auction 3B: 4♦ is natural because West East West East West East the auction started at the 1♦ 1♠ 1♥ 1♠ 1♥ 2♣ two level. Presumably East Hand 1A: 3NT. This hand is not worth 2♥ 3♣ 2NT used Stayman to look for 11 points. 4-3-3-3 shape, a spade fit and now is isolated queens, high cards Auction 2A: Game forcing. 3♣ is a showing a diamond suit. concentrated in the short new suit at the three level. Auction 3C: 4♦ is natural. 3♦ was a suits and no intermediates Auction 2B: Invitational. 3♣ isn’t a new transfer showing at least are all reasons to lower your suit. West will only bid on five hearts. East also has horizons. You have plenty of with a reasonable club fit. at least four diamonds. points for game and even Auction 2C: A sign off. East has long if you find a 4-4 spade fit diamonds in a hand too 4 Using my bidding principles, is your spades are so poor weak to bid an immediate the 4NT bid in the auctions shown that you may well find it a 2♦ opposite 1♥. quantitative or Blackwood? If it is hindrance rather than a help. Auction 2D: A sign off, though West Blackwood, what is the agreed trump Hand 1B: 3♣. Stayman. A much better has a right to proceed if he suit? 11 points. Look for the 4-4 is strong with a club fit. East spade fit. If partner has four has long clubs in a hand too Auction 4A Auction 4B Auction 4C spades I would use Blackwood weak to bid an immediate 2♣. West East West East West East and settle for 6♠ unless two Auction 2E: Invitational. West has 2NT 4NT 2NT 3♣ 2NT 3♣ key cards are missing. If shown extra strength by 3♦ 4NT 3♥ 4NT partner shows four hearts jumping to 3♥ but East’s 1♠ you will bid 3S to see if he shows no extra strength. East Auction 4D Auction 4E Auction 4F is 4-4 in the majors. If this could have just six points. West East West East West East is not the case, you will aim Auction 2F: Game forcing. East has 1NT 2♥ 1♠ 2♣ 1♠ 2♣ for 6NT. If partner rebids 3♦ shown extra strength by 3♠ 4NT 2♠ 4NT 2♦ 4NT denying a 4-card major, I changing suit at the two level. would bid a quantitative 4NT. West has shown extra strength Auction 4A: Quantitative. The previous Hand 1C: 6NT. You have between 34 by jumping in support of bid was no-trumps. and 36 points between the diamonds. Note this auction Auction 4B: Quantitative. The previous two hands so 6NT is likely allows East to bid 4♥ now and bid was conventional. to be safer than a small aim for the major suit game. Auction 4C: Blackwood with slam in spades, especially Auction 2G: Forcing for one round. hearts agreed. when you have such a West has shown extra Auction 4D: Blackwood with spades poor spade holding. strength by reversing but East agreed. West has rebid 3♠ could have just six points. when he was asked to rebid For questions 2, 3 and 4 you might Auction 2H: Game forcing. 3♣ is a 2♠. The reason is that he find it an instructive exercise to give new suit at the three level. likes spades. 3♠ is called the question to your regular partner(s) Auction 2J: Game forcing. West ‘Breaking the transfer’. to see whether they agree with you. has 15-19 points and East Auction 4E: Blackwood with spades changed suit at the two level. agreed. 2 Using my principles of bidding, are Auction 4F: Blackwood with diamonds the following auctions game forcing, 3 Using my bidding principles, which of (the last bid suit) agreed. n forcing for one round, invitational or the following auctions are cue bids? a sign off?

BRIDGE August 2020 Page 35 David Stevenson Answers Your Bridge Questions What Happens After An Illegal Call Is Condoned?

I do understand that general to the situations in a button) and tell the 4-12 points, with a you are obliged to where the call is illegal, director who will investigate. five point spread. Q explain to us the for example a double of Could it be an laws of bridge, and not to partner’s bid. The illegal ♣♦♥♠ allowed deviation? justify them, but the law double cannot stand, of We play at level 4. Would about what happens after course, but the fact that Playing social it be allowed at level 5? an illegal bid of ‘double’ an opponent condoned bridge at home Alex Mathers, (as explained in BRIDGE it by calling means Q this hand was Northallerton. 208 - April 2020, final there is no penalty. doubly memorable. question) does strike me There is no rule as particularly perverse. ♣♦♥♠ whatever about Why should the Dealer South. Love All. A the point limits to a offending side (E/W) be I was playing 3NT ♠ A 7 weak two, whether natural treated so leniently (no in an EBU online ♥ A 4 2 or as part of a convention, penalties or constraints), Q duplicate and I ♦ Q J 4 for example the Multi 2♦, when the director is was going to make ten ♣ A K 10 6 3 at Level 4, ie all events called after East’s offence tricks, but after about eight ♠ Q 8 5 3 ♠ K 9 6 4 2 except Novice and Holiday N has been compounded tricks the cards disappeared ♥ K 8 5 W E ♥ Q events. So you could play it by West’s response? and we moved for the next ♦ K 9 8 6 3 S ♦ A 7 5 2 as 3 to 11 if you wished. Whereas, if the offending round. What happened? ♣ 7 ♣ Q 5 2 If you normally play it as 5 double is called out Jackie Smith, ♠ J 10 to 9 and just on one occasion immediately, the pair are Blue Jackets BC. ♥ J 10 9 7 6 3 happen to decide to open it understandably penalised by ♦ 10 on 3 points that is a deviation a constraint on their bidding. You ran out of time. ♣ J 9 8 4 which is perfectly legal. Stephen Whitaker, There is a set amount Sherborne, Dorset. A of time for each round ♣♦♥♠ (similar to playing in a club) South opened 2♦, alerted as One of the parts of and a timer called “Clock” a Multi. They bid to 4♥ by We play a the laws that has shows how many minutes South, making (the defence variable 1NT. A stood the test of time left. But so long as most of was not optimum), with a Q Non-vulnerable with very little change is the board has been played combined twenty-one points, Kamikaze 9-11 points. the general approach that a score will be allocated which was memorable. Vulnerable Intermediate there is no penalty once an to you on this board. If ten We play a Multi, 12-15 points. illegal call is condoned, ie tricks were obvious I expect hearts only, with two I am considering a by the next player calling you were allocated 3NT+1. strong options. Woodson 2-Way No after the illegal call, whether There is a tab marked We play our weak Trump. he does so intentionally or “History” and if you go option non-vulnerable Is there any limit to the because he did not notice to there when you have a as 5-9 points, 5 or 6 continuous range of 1NT? the call was illegal. So if moment you will see all the hearts, but vulnerable Can we play non-vul 9-15 there is an insufficient bid boards you have played 8-12 points, 6 hearts. points and vul 2-15 points? and the next player passes and you can see what they A successful 2♦ Multi If so are any the insufficient bid stands: if gave you on the board. If with three points was continuations permitted? there was a call out of turn you found they had given equally memorable. Alex Mathers, and the next player bids you 3NT= then you should Is bidding a Multi with Northallerton. the call out of turn stands. press the Call Director button three points allowed? The same applies in (usually three parallel lines The blue book suggests Continued on page 38 ... u

Page 36 BRIDGE August 2020 Answers to Julian Pottage’s Defence Quiz on page 17

1. ♠ A K J 6 Unless declarer is void in diamonds, diamond return backfires. Partner has ♥ A K Q 9 4 the suit will run. Indeed, if partner has no more trumps and declarer easily takes ♦ 7 5 4 two low hearts, declarer could be in a the rest of the tricks. ♣ 8 position to run the rest of the tricks upon The point is that you know declarer ♠ 7 4 ♠ 8 5 3 gaining the lead. You surely need tricks began with at least five hearts, so ♥ 8 3 2 N ♥ 10 7 5 quickly. partner has at most one left, making two W E ♦ A Q 10 8 ♦ J 6 3 If partner has the ♠A, your side can ruffs impossible. You should thus cash S ♣ Q 10 5 2 ♣ A J 7 3 take the first three tricks with the ♣A the ♣A before trying for a diamond ruff. ♠ Q 10 9 2 and ♠A-K. That will at least cut down on If (holding the ♠A) partner plays a big ♥ J 6 overtricks. Better still, if partner has the club, you could switch to a spade instead ♦ K 9 2 ♠10 (or the ♠J) with the ♠A, you might and even score a ruff yourself. ♣ K 9 6 4 beat the contract. Since you want to have the lead for a possible finesse against the ♠J, after taking the ♣A you switch to the 4. ♠ K J 8 West North East South ♠K. When that holds, you can continue ♥ 8 1♥ Pass 1♠ with the ♠9 and your side takes the first ♦ A K 10 9 6 4 Pass 4♣1 Pass 4♠ five tricks. ♣ K Q 8 All Pass ♠ 6 5 4 ♠ Q 10 9 2 1 Strong spade raise, short clubs ♥ K Q 9 3 N ♥ A 7 2 3. ♠ K Q 10 8 3 2 ♦ 5 2 W E ♦ Q 8 3 S Partner leads the ♣2. What is your plan? ♥ Q J 5 ♣ J 10 9 6 ♣ 7 5 3 Clearly you should take your ♣A and ♦ Q J 10 ♠ A 7 3 attack dummy’s diamond weakness. ♣ 10 ♥ J 10 6 5 4 Since declarer will surely run the hearts ♠ 9 6 5 4 ♠ J ♦ J 7 to discard diamonds if given the chance, ♥ 2 N ♥ A 10 7 ♣ A 4 2 W E you need to take what you can at once. ♦ 5 S ♦ K 9 8 4 3 2 The way to do so is to switch to the ♦J. ♣ Q 9 8 6 5 3 2 ♣ A J 7 This offers the chance to hold the lead ♠ A 7 West North East South if, as you hope, partner has the A-Q-10. ♥ K 9 8 6 4 3 Pass ♦ A 7 6 Pass 1♦ Pass 1♥ ♣ K 4 Pass 3♦ Pass 3NT 2. ♠ Q All Pass ♥ A Q ♦ A K J 10 7 5 4 West North East South Partner leads the ♣J. Declarer wins in ♣ 8 6 3 1♠ 2♦ 2♥ hand with the ♣A and runs the ♦J to your ♠ A 10 5 4 ♠ K 9 8 3 Pass 3♥ Pass 4♥ ♦Q. What is your plan? ♥ J 8 2 N ♥ 10 7 5 3 All Pass A club return would be safe but no W E ♦ 8 S ♦ 9 6 3 more than that. If you are going to return ♣ Q J 9 5 2 ♣ A 7 Partner leads the ♦5, won by the ♦Q. At a club, it is right to cash the ♥A first, ♠ J 7 6 2 trick two you capture the ♥Q with the ♥A. hoping partner has the ♠A and declarer ♥ K 9 6 4 What is your plan? ends up making a losing spade lead ♦ Q 2 There is a danger of playing too quickly from dummy. Putting partner with the ♣ K 10 4 here. With the ♦4-3-2 in your own hand, ♠A, however, would mean declarer has you know the ♦5 was a singleton and bid 3NT with nothing close to a spade may thus be itching to return the suit. stopper. With 9 points assumed in the West North East South Furthermore, if you are used to giving passed hand on your left (the ♠A, ♣A 1♦ Pass 1♥ a suit-preference signal when leading a and ♦J), partner is likely to have some Pass 3♦ Pass 3NT suit for partner to ruff, you might look at good hearts. Since you may need to All Pass the ♣A as a re-entry and so lead back finesse twice against declarer, you should the ♦2, low for the lowly club suit. switch to the ♥2. This beats the contract if Partner leads the ♣Q. What is your plan? As you can see from the layout, the partner has K-J-9-x or better. n

BRIDGE August 2020 Page 37 t ... Continued from page 36 have found yourself not matter of courtesy … a and make announcements. So long as the getting a bottom. That is the player should refrain from The effect of this was to minimum is 9 or nature of duplicate bridge detaching a card before annoy the people who were A higher you may and perfectly normal. it is his turn to play.” Like never ever rude and had no play any range of any There may be occasions all laws it can be enforced beneficial effect whatever, width with any responses where what went through a by a disciplinary penalty. and when important thereto. So your 9 to 15 player’s mind may be clear However, in days gone announcements are made 1NT is perfectly legal. from a look at the hand but by when we played the players were less inclined However, the 9-15 1NT is not often. To give you an bridge in the real world, as to listen. No-one has ever not the Woodson 1NT. That example, suppose a player director I would from time stopped being rude because convention had split ranges responds to Roman Key to time remind players at some announcement so its protagonists played Card Blackwood with 5♣ the beginning of a session sometime told them not to 1NT as either 10-12 or 16-18 and wants to change it to of one or other aspect and similarly a player with and felt that the agreement 5♦. He has no aces. He is of Law 74, as many are a well-known annoying caused more problems to playing the version played simply ignorant of it. habit with his cards will their opponents than to by a large minority where For example, ‘Law 74 not change because of a their partner. However that 5♦ shows no aces (or three) deals with courtesy at the general announcement. is not legal nowadays. unlike the majority who play bridge table. I am sure you 5♦ as one ace (or four). all want to be courteous ♣♦♥♠ ♣♦♥♠ Now what has happened? but you may not be aware Has declarer miscounted that a player should not, We have been I read with interest his aces, perhaps he counted for example, detach a card playing Multi your reply to Alex the king of a suit as an ace from their hand before it is Q 2♦ on BBO. Q Mathers in the forgetting it was not trumps, their turn to play.’ Once we An American opponent June issue of BRIDGE. in which case the bid may not get back to playing bridge asked what it was? There were a number of be changed? Has declarer in the real world again, Explained as weak, minor issues in the scenario. forgotten he is playing these I plan to continue this. hearts only Multi, with 16+ If N/S allowed 13 tricks responses until just after he Sean Haffey by email. strong distributional and they deserve zero not 25%. bid, in which case the bid 23+ strong flat hands. However, my query relates may not be changed? Has While this annoying They were not sure it could to your wording ‘directors do declarer intended to bid habit would be better be played on ;an ACBL site. not look at players’ hands’ 5♦ and accidentally pulled A if players did not Can you advise us? Does this mean because out a 5♣ card, which may do it there are also a lot of Are all the competitions on they choose not to, or the be changed? The thing other annoying habits as BBO governed by the ACBL? shouldn’t or they are not that is certain is that a look well. But announcements Alex Mathers, Northallerton. allowed to? You say that at declarer’s hand will such as you suggest, in my the director should judge not tell you which, but will view, do very little good. In an ordinary ACBL whether a bid was clearly a certainly lead to arguments They tend to annoy people run online duplicate mistake, not likely, conveyed from whoever is ruled who do not do them and A their Open Chart is little extra information or against if you do look. the more announcements used, which is pretty much was a change of mind. This It is not illegal for a of any sort that people club level bridge and your judgement might not be easy director to look at the hand, make the less people listen Multi is not permitted. and would presumably lead just very bad practice. to them. Furthermore, the No, there are many, many, to a different adjudication. Competent directors never actual people who do this many competitions on BBO I cannot see how a fair do and training courses tell annoying habit will not not run by the ACBL. The decision can be arrived directors never to look. change their habits because EBU is running 35 events a at without a good look of a general announcement. week, shortly to become 42, at the offender’s hand. ♣♦♥♠ In my own club there was a the WBU runs events, various Club Player, North Wales. case where a player was rude counties and clubs run events In the May issue to another player. Instead of and I am sure that there are It does not matter of BRIDGE, Brian dealing with the player who events run all over Europe. what they deserve: if Q McLaughlan asked was rude, the committee All EBU and WBU events will A when the hand was about a player at his club decided to put up notices permit your Multi. n played they got 25% then who has “the annoying that is what they got. I am habit of pulling out the card Email your questions for David to: sure you have bid, played he intends to play as soon or defended badly and got as dummy hits the table.” [email protected] an awful result but because This is covered by Law Please include your postal address of the results at other tables 74. 74B3 states “As a

Page 38 BRIDGE August 2020 Answers to David Huggett’s Play Quiz on page 17

You are declarer in 3NT after West has singleton ace of hearts. In the latter case, 1. ♠ J 7 3 2 overcalled in hearts. West leads the ♥7 you discard another club from dummy ♥ K J 10 6 and East plays the ♥10. How do you plan and duck a heart establishing the king ♦ 10 6 the play? for your twelfth trick. All that is required ♣ 9 6 5 You need to make four diamond tricks is a watchful eye. ♠ 9 8 5 4 ♠ 10 6 in order for this contract to have a happy ♥ 4 N ♥ Q 3 2 outcome; of course it is imperative that W E ♦ A 9 8 2 S ♦ 7 5 4 3 East never gains the lead for fear of a 4. ♠ K 7 5 ♣ A K 8 4 ♣ Q 10 7 3 heart back to West, who would cash at ♥ A K 5 ♠ A K Q least four tricks in that suit. So the normal ♦ K 9 7 5 ♥ A 9 8 7 5 play of taking the ace of diamonds and ♣ K 6 4 ♦ K Q J then playing to the jack looks as if all the ♠ A J 10 8 3 2 ♠ 9 4

♣ J 2 requirements are met. Almost but not ♥ J 10 9 2 N ♥ Q 7 6 4 3 quite. On a bad day, East would start with ♦ 8 W E ♦ 6 4 S Q10xx in diamonds and this plan would ♣ Q 2 ♣ J 10 8 3 You are declarer in 4♥, having opened fail; if instead you started by cashing the ♠ Q 6 2NT in fourth seat. West leads the ace, king ace and playing a low diamond to the ♥ 8 and a low club. How do you plan the play? nine you would always succeed unless ♦ A Q J 10 3 2 With a certain diamond loser to go with West started with four diamonds to the ♣ A 9 7 5 the two tricks already lost, it is imperative Q-10. that you locate the queen of trumps if you are to succeed in your contract. In You are declarer in 6♦ after West made a isolation the correct way to play the heart 3. ♠ A 7 5 weak jump overcall of 2♠. West leads the suit would be to play for the drop of the ♥ 6 5 ♥J. How do you plan the play? queen in two rounds and maybe that ♦ K Q 7 It looks as though you have a certain is what you have to do here. However, ♣ A Q 7 6 5 loser in spades and another in clubs but it cannot cost to do a little preliminary ♠ 6 4 ♠ 10 9 8 3 things are not always as they seem; here work first and here it would be expedient ♥ A Q J 10 9 4 N ♥ 8 3 2 it is possible to use the knowledge that to play on diamonds first, merely to see ♦ 10 W E ♦ 8 6 5 4 3 West most likely has the spade ace. Win S who has the ace. If it is West, then he ♣ J 10 9 8 ♣ 4 the opening lead in dummy, draw trumps cannot have the queen of trumps as that ♠ K Q J 2 ending in hand and lead your low spade would give him thirteen points and he ♥ K 7 towards dummy. has already passed, so you could take ♦ A J 9 2 If West rises with the ace, you have two the finesse against East knowing it is ♣ K 3 2 tricks in the suit and can discard two bound to work. But if East happens to losing clubs from hand on the remaining hold the diamond ace then you should top heart and the spade king. play for the drop in hearts. You are declarer in 6NT after West If West follows with a low spade you opened a weak 2♥. West leads the ♣J. win in dummy, cash the other top heart How do you plan the play? throwing the spade queen from hand 2. ♠ K 9 7 5 The contract is a sound one and is and eventually you can ruff your last club ♥ 8 6 clearly laydown if clubs break 3-2. in dummy. n ♦ A 5 4 Strangely though, the contract is equally ♣ A 9 3 2 laydown if West started with four clubs To support Little Voice, send your ♠ J 10 3 ♠ Q 8 6 and presumably the ace of hearts. Win used stamps to: ♥ A Q 9 7 2 N ♥ 10 5 3 the lead in hand and immediately play Malcolm Finebaum W E ♦ 2 ♦ Q 10 8 7 back another club to test the situation. Flat 8 Mountford House S ♣ Q 10 8 6 ♣ J 7 4 If West shows up with a four-card suit, 8 Crescent Road Enfield EN2 7BL ♠ A 4 2 cash the four spades throwing a club Little Voice work directly with an ♥ K J 4 from dummy and now run the diamonds. impoverished urban community to support ♦ K J 9 6 3 When you play your last diamond the economic development of women, to provide primary education to children and ♣ K 5 master, West will either have to unguard vocational opportunities to youth. the clubs or more likely come down to a www.littlevoicefoundation.org

BRIDGE August 2020 Page 39 Catching Up with Sally Brock Seven Days

here is not a lot to say about by Sally Brock this that is any different from last month. We continue to play Tquite a lot with friends – Skyping at the TUESDAY MONDAY same time and generally being sociable. The Lockdown League has just finished Monday is one of my busiest days in the Tuesday is busy too. At 10am I have an – that is eight-board matches four Lockdown. First up is an online game hour’s lesson with Fawzi. He has been times a week, followed by a post- with my American-friend-living-in- playing since he was seven years old mortem in Hang Out afterwards. We Sydney Karen, my Austrian-friend- and is now in his eighties, but it is a did OK – comfortably in the upper half living-in-Vienna Jovi and my English- pleasure to spend time with him and of the table but never showing any real friend-living-up-the-road, Susanna. I hope I am increasing his pleasure sign of winning. We are still playing in This week I play with Susanna and in the game. Then I meet up online the Young Chelsea Super League. This we are completely outclassed, not with Tom. I am playing with him in league started back in January and is surprisingly, given what good players a tournament next week, and we need now drawing to its close. At present we Karen and Jovi are. Then at two to agree our system and have a bit of are lying second but quite a way behind o-clock I am online with Debbie (I’ve bidding practice. That seems to go OK the leaders. been teaching her online for over ten and then at four o’clock I have an hour On the domestic front, I am still years now) and Kath. I play against with another new client, Andy, who is working on my peacock – though not them for a couple of hours. It’s always a pretty decent player. so confident it is going to turn out well. good fun and it is surprising just how As soon as that’s finished I go for I have been having problems with my much comes up. At 4.30pm I have an a quick walk in Holland Park with sewing machine. I had this problem online practice session with Fiona – Susanna – all at a discreet two metres before: it is happily zigzagging away with David, our coach. We have just apart (well, sort of). It is so nice to see but suddenly stops and changes to a decided to stop playing a strong no- real friends in real life, rather than just straight stitch. This first happened a trump and two-over-one, and instead the camera version. few months ago, before the Lockdown, play my favourite weak no-trump with At six it is Hanna. She is one of my and I paid for a full service but that five-card majors. I am so pleased about U26 women’s squad, though actually didn’t improve things. I ended up this because I really don’t like the two- she is now too old.If the 2020 World thinking it must have been a particular over-one methods seemingly beloved Championships go ahead (maybe in fabric that the machine didn’t like by most of the bridge world. However, 2021) then I am hoping she will play and unpicked lots of it to replace that it is a very different approach and with Siyu, who is just finishing off fabric. However, the problem has we need to practise a lot. Then at six exams. So, for the moment I am just recurred. I emailed the manufacturer o-clock is the first match of the new playing with Hanna (and later will of the machine and they thought that season of the Lockdown League. The play with both of them), often against probably some interior parts of the matches are of just eight boards. We other junior pairs, but today just machine had worn out in the 30 years start by playing last season’s winners against robots. The first board goes or so I have had it. The time has come and lose. The main swing is a 3NT swimmingly: to buy a new machine. We are now with 14 points opposite 11. We bid a allowed to meet with a single friend for weak 1NT – 3NT; they bid 1♣ – 2NT a walk in the park. I have been doing – 3NT. The five-card major on lead Dealer East. N/S Game. this with Margaret – in Hyde Park. The against us is the one to beat it; the five- ♠ K 10 9 7 outdoor restaurant is serving drinks, card major on lead against them is the ♥ Void sandwiches and ice cream, so we can one to let it through: unlucky. After ♦ A Q 9 7 6 3 meet up, get a drink and then find a the boards we meet up on Zoom for ♣ A K Q park bench to sit at opposite ends of. It a post-mortem. And that is the end of ♠ Q 5 4 ♠ J 6 3 feels really strange not to hug. my bridge for the day. The guy in the ♥ A K 10 7 6 3 N ♥ Q J 9 5 4 W E A friend told me that she has booked flat across from me has had a barbecue ♦ K 10 8 S ♦ J 5 4 2 a holiday, in Austria, at the end of July. in our garden and brings us four huge ♣ 2 ♣ 7 What an exciting thought. I’m not sure skewers of cooked chicken and lamb. ♠ A 8 2 that I’m quite ready to do that yet, but So I shelve my plans for fishcakes and ♥ 8 2 I can spend plenty of time thinking have meat and salad instead. Then it is ♦ Void about where I might go. The Italian time for TV – Briony and I have been ♣ J 10 9 8 6 5 4 3 Lakes are appealing. n watching Hinterland recently.

Page 40 BRIDGE August 2020 After East passes, I have to decide what else, so here I decide to open 3♥. The her (it is one of the children’s birthdays to open. I like to open at the four level auction progresses rapidly (the wine soon). So I do that. We bought a soft- when I have an eight-card suit but am is flowing freely by now). West leads a top little Fiat earlier in the year and not sure that I really have enough for diamond; I win in hand to play a spade now it is coming into its own as we a 4♣ opening at adverse vulnerability. to the jack and queen. A diamond can take the roof off and drive around Oh well … nothing ventured … So, comes back and all I can do is win in (only a little!) in the sunshine. I get this is the auction: dummy, ruff a spade, try the ♥K, pick home in time for my second session up West’s queen and when the spades of the week with Fawzi. Then I do my West North East South break I make my contract: +1620. The exercise bike, colour my hair (that’s the Pass 4♣ redouble was just the icing on the cake, first time for a while – I’m desperate 4♥ 5♥ Pass 5♠ but satisfying nevertheless. for a professional cut), have a shower, Pass 7♣ All Pass have some lunch on my balcony all in time for a practice session against John When partner bids 5♥ she is showing WEDNESDAY and Jackie. We’ve been trying to find a really good hand with hearts sewn Another new client at 10 o’clock, a way to communicate while we play up. Although I am worried about the Neville, a lovely man who seems to but they are even greater technological quality of my trumps I do not feel I really enjoy our sessions. Then I go for dinosaurs than we are and everything can refuse to show her a first-round a walk up Kensington High Street with we try fails. After our session, I leap spade control. Consequently I cue-bid Briony. We go to Waitrose for some into the car and go to pick up Briony. 4♠ after which she bids ♣7 – it was just bits and pieces and buy a delicious- We then spend the evening watching what she wanted to hear. There aren’t looking tray of strawberries from a movie. any problems in the play. the greengrocers over the road. Then I just have time to cook my dinner the chemist to pick up a prescription (fishcakes) before we are online against before going home for another practice FRIDAY Frances and Graham. This is my session with Tom. This time we are I’m up early to get all my exercise and favourite regular session. Somehow dealing with competitive bidding. ablutions done in time to start a two- they play at our sort of pace and we We talk a while and bid a bit and hour practice session with Tom at can discuss at our sort of level without everything seems to be going OK. I ten. We play against Barry and Steve, having to explain the obvious. We get have a bit of free time in the afternoon and lots of interesting stuff comes through 48 boards in all. This is my and then at six o’clock it is the second up. I wouldn’t say we actually do favourite: match of the Lockdown League and very well, but we see eye to eye most this time I am playing with Frank. We of the time, which is what matters. bid three slams – one is flat, on one we Then I get my stuff together and walk Dealer South. Love All. gain (we get lucky with the opening to meet Margaret in Hyde Park. This ♠ K J 8 7 5 4 lead), and the other is also flat because is the third week we have been doing ♥ A 6 5 2 of a 5-0 trump break. It is all going this and it is great to see a real friend. ♦ A 9 8 OK until the last board; we are mildly We have a coffee and some lunch on a ♣ Void wimpy in the auction and sell out to park bench, and then I leave to walk ♠ 10 9 3 ♠ A Q 2 an unbeatable 2♠, while in the other to Barry’s (via my favourite ice-cream ♥ Q 9 4 N ♥ Void room the opposition with our hands place). I arrive about 3.30pm in time W E ♦ J 10 ♦ Q 7 4 3 2 S press on to 2NT which is let through for my coaching session with David ♣ 10 9 8 6 3 ♣ A K Q J 4 by our team-mates. That is enough to and Fiona. We are working on how ♠ 6 lose the match by a few. These eight- to bid after partner makes a take-out ♥ K J 10 8 7 3 board matches can be on a bit of a double at the moment. That all goes ♦ K 6 5 knife-edge. Later we go through the OK and we finish before five, enabling ♣ 7 5 2 hands as usual and then my team me to set up my teaching session with have a match in the Young Chelsea four Irish women. It is quite a skill to Super League. I would usually watch get them all together, both on BBO West North East South my team when we are not playing but and three of them on Zoom while one 3♥ today I completely forget about it and is on Skype. None of them have been Pass 6♥ Dbl Pass watch a movie instead. To be honest, it playing very long, but they are keen Pass Rdbl All Pass is quite a relief to be away from BBO and seem to enjoy the sessions – one for a while. of them is only thirteen years old. That I like to open a Multi with a five-card finishes at six when Barry is starting his major in first seat non-vulnerable. That eight-board match in our Lockdown lets me interfere with my opponents’ THURSDAY League. So I watch that and join the auction more often. So when I get a Briony is doing a day’s nannying and Hang Out for the post mortem. We win fairly perfect weak two hand I need has asked me to give her a lift there narrowly – our first win in this new to think about doing something because she has so much to take with series (only three matches so far). u

BRIDGE August 2020 Page 41 t Then steak and kidney pie with play against Claire and Paul. Our very lots of vegetables for dinner. And, first deal is instructive: Dealer East. Game All. thankfully, an evening with no bridge. ♠ J 6 ♥ A K 5 3 2 Dealer South. Love All. ♦ K 7 SATURDAY ♠ A ♣ A Q J 2 Up early for training with Fiona at ♥ Q J 10 9 ♠ 9 5 3 2 ♠ K nine. I hope our is not ♦ K Q J 10 6 ♥ Q 10 8 6 N ♥ J 7 ♣ A J 9 ♦ Q 6 3 2 W E ♦ 9 8 4 getting too complicated. I have another S practice session with Tom and then ♠ K Q J 6 2 ♠ 10 7 5 ♣ 10 ♣ K 9 8 7 6 5 3 (after a delicious beef stir-fry dinner) ♥ A 5 4 2 N ♥ 7 ♠ A Q 10 8 7 4 W E we play online against Norman and ♦ 4 S ♦ A 9 8 5 2 ♥ 9 4 Kay. This is good fun – skyping at the ♣ 7 5 2 ♣ Q 10 8 6 ♦ A J 10 5 same time, with glasses of wine. ♠ 9 8 4 3 ♣ 4 ♥ K 8 6 3 ♦ 7 3 SUNDAY ♣ K 4 3 West North East South In the morning we play for a couple of 3♣ 3♠ hours against Karen and Alex who are Pass 4♣ Pass 4♦ in Sydney. It is a bit weird because they West North East South Pass 4♥ Pass 4♠ are mellow after a good dinner and Pass Pass 5♠ Pass 6♠ a couple of glasses of wine, while we 1♠ Dbl 2♠ Pass All Pass have only just got up. Still, it is fun as Pass Dbl Pass 3♥ usual – though we are a bit sceptical Pass 4♥ Dbl All Pass I think we have a pretty decent about their relay system, while I am sequence. I nearly bid more over 4♥, but sure they are not very impressed with I don’t really have an opening bid in Barry is a very optimistic slam bidder, our slap-bang-wallop approach. Then second seat, but I fancy my majors and so I sign off rather conservatively. we have a quick lunch and go out in the my good spade suit. When partner When he bids one more I am happy to car. This is a major event in our lives decided to double the final contract cooperate. I am not so good in the play, after weeks of lockdown. We go out of I thought I might live to regret the however, and make it more difficult London to Black Park Country Park. opening. I did, but I shouldn’t have. I for myself than necessary. I win the It is a lovely place near Slough with a lead a top spade and see a depressingly club lead in the dummy and rather huge wooded area and a big lake in strong dummy. However, if we keep carelessly lead the ♠J from the dummy. the middle. However, like many great playing spades maybe declarer will If I had led a small one I would have pubs, it would be fantastic if it wasn’t lose trump control. Declarer wins the been able to claim; as it is … when the for all the other people. It is teeming. ace and plays a couple of trumps from second round of trumps sees East (not After waiting for ten minutes or so in the dummy, both of which I duck. unexpectedly) show out, I play two top what seems to be a very slow-moving Now he plays a top diamond, which hearts, and then play the ♣Q from the queue to pay for a parking ticket we partner wins and switches to a club dummy, covered and ruffed by me, decide to risk it and do without it (a rather than the spade necessary to beat West pitching a diamond. good decision). We walk around the the game. Of course, it was reasonable Next, I play two more rounds of lake and watch other people with dogs for him to place me with the ♣K for trumps. West wins the second round and kids swimming in the lake. Then my bidding. Because the winning and plays the ♥Q, which I ruff. Having back home to recover. I make a Zoom defence was completely obvious to found out that hearts are not 3-3, it is conference call with my late sister’s me, I hadn’t realised that it might be just a question of who has the ♦Q. I family. She died ten years ago; her less so for him. I had two chances am inclined to play East for it, because husband (now remarried) and their to make suit-preference signals in West voluntarily pitched a diamond three wonderful grown-up children all trumps. I actually played, extremely when she didn’t need to. But whoever Zoom together on a Sunday evening. lazily, the two followed by the four, has it, I need to cross to dummy’s ♦K So I gatecrash their meeting and catch suggesting something in clubs, when and cash the ♣J: either I squeeze West up with all their news. Later on we I should have played the five and the in the red suits or else I can finesse four, suggesting nothing in clubs and through East. West takes too long Write to Mr Bridge at: therefore that he continue spades: my to discard a diamond on the ♣J, so I Ryden Grange, Knaphill, fault. It is important to help partner change my mind and play her for the with suit-preference signals whenever ♦Q. Whew, that could have been a lot Surrey GU21 2TH you can. easier. or e-mail: Things improve after that and well When we are finally finished it [email protected] past midnight the following hand is 1.15am. Time goes quickly when turns up (transposed for convenience): you’re having fun. n

Page 42 BRIDGE August 2020 Mr Bridge Luxury Answers to Bernard Magee’s Playing Cards Bidding Quizzes 1-3 on page 7

1. Dealer East. Love All. you may well have intended to bid 2♣ ♠ 7 6 3 ♠ A 9 8 4 2 yourself. Generally, when this is the ♥ A 3 2 N ♥ K Q 5 case, your correct action will be to pass W E ♦ K 9 8 7 6 S ♦ A because you may well harbour hopes of ♣ 8 7 ♣ 9 5 3 2 a penalty. Remember that doubles of suit bids at low levels are for take-out, so you cannot double yourself. However, with West North East South four clubs in your hand and six often with 1♠ 2♣ the overcaller, your partner is likely to be ? short and may well reopen the auction with a double, which you can then pass. 2♠. Just 7 HCP so passing has to be Your ♣9 is an important card in this an option, but it is always nice to make respect as it gives the potential for a third a call if you have 6+ points just to aid trick in the suit. your partner in the auction. You might As you can see, if you pass and North contemplate a double, but it is better to passes, then East is likely to double for use that when you have at least four cards take-out, expecting to play in any of in the unbid major. You are too weak for the three other suits, but being happy a 2♦ bid – that should still be showing enough when you pass, because he 10+ points (or 9 at a stretch). There is trusts your judgement. one more option and that is supporting to 2♠. You have only 3-card support but in a competitive auction striving to 3. Dealer East. Love All. support is so important. ♠ 7 6 ♠ A K 5 4 2

It is particularly common on hands ♥ A Q 9 8 7 4 N ♥ 5 2 where your possible 1NT response has ♦ 6 5 W E ♦ A Q 8 2 S been taken away by a 2-level overcall. ♣ 9 3 2 ♣ 7 6 In this auction, East should not be surprised that you have just 3-card support. Do not worry about the low West North East South cards either – if you have an 8-card fit 1♠ 2♣ then 2♠ will do fine, and even a 4-3 fit ? should play OK as you can expect your partner to have a balanced hand too Dbl. Surely the answer is 2♥ you might Box of 72 - £72 strong to open a weak NT. say? The problem with 2♥ is that it shows a stronger hand and your partner (36 red-backed, 36 blue-backed) is obliged to bid again, so you end up Strip of 12 - £19.95 2. Dealer East. Love All. going past the contract you would like to (6 red-backed, 6 blue-backed) ♠ 7 6 ♠ A K 5 4 2 finish in. ♥ 9 4 2 N ♥ A Q 7 6 A double in this auction is often called ♦ A 7 6 5 W E ♦ 9 8 2 a and this is the type of Includes postage to S ♣ A Q 9 2 ♣ 6 hand from which that name derives. If you UK addresses. double and bid your suit you are showing a weak hand (negative), whereas if you Cards are not bar coded. West North East South make an immediate suit overcall you are 1♠ 2♣ showing a stronger hand. ? Here, you double, partner rebids 2♦ ( 01483 489961 and then you bid 2♥ and your partner can Pass. The same overcall as question 1, pass this, leaving you at a comfortable www.mrbridge.co.uk/shop but a slightly different disruption because level. n

BRIDGE August 2020 Page 43 BERNARD Letter From Overseas by John Barr MAGEE’S TUTORIAL CD-ROMs Which Slam

ADVANCED To Bid? DECLARER PLAY l Overtricks in No-trumps l Overtricks in Suit Contracts l Endplays l Avoidance l Wrong Contract £81 l Simple Squeezes uring a recent pairs game I the auction had been: l Counting the Hand picked up the strongest hand l Trump Reductions & Coups I’ve seen for some time and, West North East South l Playing Doubled Contracts Dwhile I was wondering how to bid the 2♠ l Safety Plays hand, my partner opened a weak 2♠ (5 Pass 2NT Pass 3♣ to 9 points). I held: Pass 6NT All Pass ADVANCED ACOL BIDDING The full hand was: ♠ K Q 10 7 l Basics ♥ A K l Advanced Basics ♦ A Q J 8 ♠ K Q 10 7 l Weak Twos £96 ♣ ♥ l Strong Hands A 5 2 A K l Defence to Weak Twos ♦ A Q J 8 l Defence to 1NT ♣ A 5 2 l Doubles My first thought was that partner ♠ Void ♠ 9 8 2 l Two-suited Overcalls didn’t need a lot for seven spades to ♥ J 8 6 4 3 2 N ♥ Q 10 7 5 W E l Defences to Other Systems ♦ K 10 7 6 ♦ 9 2 be laydown, so I started with 2NT, S l Misfits and asking for more information about ♣ 9 4 3 ♣ Q J 8 6 Distributional Hands his hand. With a minimum opening ♠ A J 6 5 4 3 partner would rebid 3♠, with a solid ♥ 9 FIVE-CARD MAJORS suit he would bid 3NT, and with any ♦ 5 4 3 & Strong No-Trump other upper range opening he would ♣ K 10 7 l Opening Bids & Responses bid a suit where he held a high card. l No-Trump Openings His response was 3♣, showing 8 or l Support for Partner 9 points and an honour in clubs – With the diamond finesse working, all l Slams & Strong Openings usually the ace or king. thirteen tricks were available in spades l Rebids If he had responded 3♠, showing a or no-trumps, but no-one bid a grand l Minors & Misfits £89 minimum hand, I would have subsided slam. Indeed, some pairs subsided l Pre-empting ♠ ♠ l Doubles in 6 . If he had shown an upper range in game, while most played in 6 , so l Overcalls hand with the diamond king, I could 6NT plus one was a top score. l Competitive Auctions count thirteen tricks (six spades, two With a hand like this it is very hearts, four diamonds and the club tempting to leap to 6♠ over partner’s Operating system requirements: ace), so I would have bid 7 no-trump. 2♠ opening, as a score of +980 for six Windows XP - 10 With the super spade fit it was spades bid and made is a big number. or Mac OS 10.08 -10.14 (Advanced Declarer Play only) tempting to bid 6♠, but that would But in pairs scoring the important be the wrong bid at pairs. We had issue is not how big a score you make, ALSO two stops in every suit, I could count but how your score compares to what l Acol Bidding 11 tricks off the top (six spades, two other players have achieved with the l Declarer Play hearts, a diamond and two clubs), same cards. l Defence See p34 and I could guarantee to develop the So scoring +1020, just ten points ( 01483 489961 twelfth trick in diamonds even if the more than the pairs who bid 6♠ and www.mrbridge.co.uk/shop finesse failed – so I bid 6 no-trump. made an overtrick, will give you most At Love All, with South as the dealer of the matchpoints. n

Page 44 BRIDGE August 2020 Answers to Bernard Magee’s Bidding Quizzes 4-6

Bernard Magee Bridge on page 7

Bernard Magee’s new subscription website includes: When one side says they are weak, then 4. Dealer East. Love All. bids by the other side are designated as ♣ Seminars ♠ 4 3 2 ♠ A 7 strong because most of the time they will ♥ 4 3 N ♥ A 9 6 5 be. W E ♣ Set hands ♦ A Q 7 2 ♦ K 8 6 4 Here, if you bid 3♥, your partner is S ♣ Quizzes ♣ A J 6 5 ♣ K Q 2 going to bid again and you will get too high. You might wonder whether you ♣ Interesting hands should double, as in Q3, but here both ♣ West North East South majors are unbid and you would not Live online events 1♥ 3♠ want to hear 4♠ from partner. Instead, ? you pass and your partner may well pass A great mix of fun, too. learning, entertainment Dbl. How high do you play your take-out doubles? and good company. I really do recommend playing 6. Dealer East. Love All. them up to at least 3♠, otherwise the ♠ A Q 8 7 6 ♠ J 2 disruption caused by pre-empts can be ♥ A 9 7 5 4 N ♥ K Q 3 too difficult to handle. If double is not ♦ 4 2 W E ♦ 5 3 S for take-out here, you do not really have ♣ 3 ♣ A K Q 7 6 5 a good option and might have to pass. Your partner would probably pass as well. Instead, your double of 3♠ allows West North East South your partner to have a go at 3NT with 1♣ 3♦ his spade stopper and the right game is ? Special Offer for reached. BRIDGE subscribers 3♠. Just ten high card points, but with £11.99 per month the pre-empt on your right, it is not 5. Dealer East. Game All. unreasonable to hope that your partner Founder Membership ♠ 6 2 ♠ Q J 8 3 might have at least three cards in one ♥ K Q 9 8 4 3 2 N ♥ 7 6 of your majors. If you do have a fit, extended until 31 July ♦ 7 5 4 W E ♦ A 2 game is likely to be on, so you should 2020 for BRIDGE ♣ 3 S ♣ A K 8 6 2 aim to show both majors. To show two suits, you do best to bid the higher suit subscribers only. Email first because when you bid the lower for details. West North East South suit on the next round your partner can

1♣ 3♦ give preference to your first suit without Normally £13.99 per ? raising the level. You bid 3♠ over 3♦ and month. then over partner’s 4♣ you bid 4♥, which Pass. A lot of players go wrong with this your partner should pass. email type of hand. The guideline that helps is Double would be for take-out, but [email protected] ‘never pre-empt a pre-empt.’ What does partner will only bid a major with four that actually mean? cards, so here he might bid 4♣ or even Visit us at: It means that when the opponents 5♣. Over 4♣ if you rebid in one of the bernardmageebridge.com show a weak hand with a long suit, majors your partner might put you with you should not bid against them with a one major longer than the other, so you This offer is only open for current similar type of hand. The reason why is do better to show your suits one at a subscribers to BRIDGE that partner will assume you are strong. time. n

BRIDGE August 2020 Page 45 Julian Pottage Answers Your Frequently Asked Questions How Should Suit Quality Affect My Rebid?

he strength of a suit can affect six-card suit. The same considerations West North East South many bidding decisions, apply to this auction: 1♥ Pass 2♦ Pass overcalls and pre-empting ? Tperhaps being more typical examples, West North East South though it certainly can affect what 1♥ Pass 1♠ 2♦ You cannot rebid 2♥, non-forcing, opener chooses to rebid. ? with 16 points and you cannot jump to 3♥ with a five-card suit. You must choose between 2NT and 3♣. Hand A Hand B Hand C Hand D With Hand E, you have a very good ♠ A K J 8 5 3 ♠ J 9 7 5 4 3 ♠ 5 3 ♠ 5 3 club suit, well worth showing, and you ♥ 7 ♥ 7 ♥ K J 7 4 3 ♥ K J 7 4 3 2 should rebid 3♣. ♦ J 8 5 3 ♦ A K J 2 ♦ 8 3 ♦ 8 3 With Hand F, while the club suit is ♣ K 3 ♣ K 3 ♣ A K J 3 ♣ A K 3 biddable, the suit is not great. A 2NT rebid describes the overall nature of your hand better. You might also West North East South With Hand C you would have rebid have noticed a slight difference in 1♠ Pass 1NT Pass 2♣ without the overcall. You cannot the quality of your spade stopper. ? bid 2♣ now because that would be Facing 10-x-x, A-x is a single stopper insufficient and a 3♣ rebid, raising while A-J gives you a double stopper. Hand A has two of the top three the level, would show a better hand Furthermore, facing Q-x-x, you would honours in spades but none in than this. You should simply pass. rather have partner declare when you diamonds. With Hand B it is the Partner will work out that you would have A-x. By contrast, if you have A-J other way round. With Hand A, I probably have rebid 2♣ or 2♦ without and are facing 10-x-x, you want the would forget about the diamond suit the overcall, more likely the former. lead coming up to rather than through and simply rebid 2♠. With Hand B, With Hand D, you should rebid your A-J. rebidding 2♦ to offer an alternative 2♥. Since you are bidding only at the denomination is normal. Partner will level you would have bid at without be aware that spades is a major and intervention, you are not promising Hand G Hand H offers the potential for a higher score any extra strength. You are, however, ♠ K 4 3 ♠ K 4 3 so will usually put you back to 2♠ with indicating a six-card suit. The length ♥ K Q 10 9 3 ♥ J 8 5 4 3 two spades and three diamonds. of the suit you are thinking of bidding ♦ 9 3 ♦ K 3 Let us change the auction slightly: is often as much as or more important ♣ A J 2 ♣ A Q 2 than its strength. West North East South 1♠ Pass 1NT 2♥ Because a change-of-suit response ? Hand E Hand F by an unpassed hand demands a ♠ A 2 ♠ A J rebid from opener in an uncontested Bidding 2♦ is no longer an option. You ♥ K J 8 4 2 ♥ K J 8 4 2 auction, rebids and opening bids are should rebid 2♠ on Hand A or Hand B. ♦ 7 3 ♦ K 3 interconnected: what you can sensibly When you are rebidding your own suit ♣ A K J 2 ♣ K J 8 2 rebid can affect your choice of opening in competition (after the fourth player bid. With the strong five-card heart has bid), you more or less guarantee a suit in Hand G, you are quite happy

Page 46 BRIDGE August 2020 to open 1♥ and rebid 2♥. With the poor heart suit in Hand H, you would Answers to Bernard Magee’s feel distinctly less comfortable about bidding the suit twice – sometimes Bidding Quizzes 7-9 partner will pass and you would not want to have this as a trump suit facing a possible singleton. If you did on page 7 open 1♥, you would probably raise a 1♠ response to 2♠ or a 2♣ response to 3♣, rebidding the hearts only after a 2♦ response. Anticipating these 3NT. Running minors are wonderful for issues, you should avoid the problem 7. Dealer West. Love All. no-trump contracts. If you bid no-trumps, by opening a weak 1NT if you can. ♠ 4 3 ♠ Q J 6 you can expect eight tricks from your If you have the strength for an ♥ K 8 7 N ♥ A 6 5 3 hand on a spade lead – your partner ♦ A K 3 W E ♦ 8 7 6 has made a call of his own, so can surely opening bid and partner enough to S respond, directing a possible opening ♣ A K 9 7 6 ♣ Q 4 3 provide another trick or two. Bid what lead or judging how high to compete you think you can make: 3NT. will not generally be at the forefront of On a bad day, North leads a heart, his your mind. Nevertheless, if partner’s West North East South partner has the ace and then switches to response shows a relatively weak hand 1♣ 1♠ Dbl 2♠ spades. However, on most normal days and both opponents are bidding then ? you will make 3NT and often you will this can be a factor. make an overtrick. Dbl. Your partner’s double is for take-out and should promise at least four hearts. Hand I Hand J With 17 points you need to do something 9. Dealer West. Love All. ♠ 5 ♠ 5 – your partner has shown some strength ♠ 4 3 ♠ 9 8 6 ♥ A Q 10 4 3 ♥ J 8 7 4 3 2 and your side owns the deal. However, ♥ K 8 N ♥ Q J 3 2 W E ♦ ♦ ♣ ♦ ♦ K 9 3 A 9 you do not have any clear action: 3 K 8 3 2 S A 7 ♣ A J 4 3 ♣ K Q J 3 would show length rather than strength ♣ A K 8 7 6 ♣ J 9 4 2 and you have no other four-card suit. With no denomination satisfactory, West North East South you should double, a responsive style West North East South 1♥ 1♠ 2♥ 3♠ double. You are showing strength and 1♣ 1♠ Dbl 2♠ ? asking partner to do something. ? East would bid 2NT here, with his With Hand I, you would pass if you spade stopper and you can then raise to Pass. Your partner’s double is not like are vulnerable (and possibly non- 3NT. a normal take-out double – it tends to vulnerable too, depending upon focus on the unbid major – so doesn’t various factors, such as how likely guarantee support for an unbid minor. partner is to have bid 2♥ with three 8. Dealer West. Love All. For this reason, with a minimum opening hearts) but if you do bid then you have ♠ K 3 ♠ 6 2 hand, you should not make a bid – 3♦ no reason to eschew a simple 4♥ bid. ♥ 8 7 N ♥ A 9 4 2 will take the auction too high, when your W E ♦ ♦ With the weaker but shapelier Hand A 4 3 S K 7 6 5 partner does not have the suit. J, you have more reason to think that ♣ A K Q 8 7 6 ♣ 5 4 3 You pass and your partner then bids 3♠ or 4♠ is making and that North 3♣, reverting to your known fit – he could be bidding on. You should doubled on the first round to see if you therefore bid 4♣. This is not a slam West North East South also had a heart fit. Clearly, if you do try (as it would be in an uncontested 1♣ 1♠ Dbl 2♠ rebid 3♦, your partner would be pushed auction). You are suggesting that a ? up to 4♣, which could well be too high.n club lead might be a good idea and perhaps that if you have a two-suit fit it could be a good idea to bid higher (a sacrifice in ♥5 over 4♠). n Mr Bridge 2021 diaries Only £14.95 each or 10 for £55 & pro rata. Email your questions With a ball-point pen and soft Kidrell cover in a for Julian to: choice of colours; navy blue, ruby red or bottle green. [email protected] ( 01483 489961

BRIDGE August 2020 Page 47 A Sally Brock Looks At Your Slam Bidding Sally’s Slam Clinic

Where did we go wrong? The first three bids are fine. Without it is perhaps a bit of a wild punt. any specific way to agree diamonds This deal was sent in by Helen McNeil (such as inverted minor-suit raises West North East South of Eastbourne. Although she and which are loved by some tournament 3♣ Dbl her partner bid to the right contract, players) 3♣ was a good first shot. Pass 4♣ Pass 4♦ she felt they could have had a better But when South showed her second Pass 5♣ Pass 5NT sequence: suit, why would North launch into Pass 7♦ All Pass Blackwood, especially with no spade control? A simple 4♦ would have been South’s take-out double on the first Dealer East. N/S Game. fine. Then a couple of cue-bids should round is clear. At this vulnerability, ♠ 9 5 have ensued: 4♥ by South, then with North’s choice of bids is between 3NT ♥ K Q no spade control but still with slam and a 4♣ cue-bid. Although he has ♦ A J 7 6 3 interest North bids 5♣. This clearly plenty of club stoppers, his hand is ♣ A K 10 9 denies a spade control and insists South too good for 3NT. When South bids ♠ Q J 10 7 ♠ K 8 6 bid a slam with such a control. With 4♦ rather than a major, it is very good ♥ J 9 6 5 4 N ♥ 10 3 ♠ W E the A, South shows it with a bid of news for North. He continues with a ♦ 8 4 S ♦ 9 5♠ and North can bid the grand slam. second cue-bid, agreeing diamonds ♣ 3 2 ♣ Q J 8 7 6 5 4 The whole auction would be: and suggesting slam values. South ♠ A 4 3 2 should definitely accept that try, ♥ A 8 7 2 West North East South holding both major-suit aces, decent ♦ K Q 10 5 2 Pass 1♦ trumps and a void club. If he had just ♣ Void Pass 3♣ Pass 3♥ one major-suit ace then he would bid Pass 4♦ Pass 4♥ it, so when he bids 5NT instead, that Pass 5♣ Pass 5♠ should show both suits controlled. They bid: Pass 7♦ All Pass That is all North needs to know in order to bid the grand slam. West North East South At some tables, East opened 3♣ Pass 1♦ and Helen asks how the hand Slam of the month Pass 3♣ Pass 3♥ should be bid then. It is much more Pass 4NT Pass 5♥ difficult, but I think this would This month’s award goes to my Pass 7♦ All Pass be a reasonable sequence, though Austrian friend Jovi Smederevac who

There has been a huge increase in bridge being played online. The excellent BBO (www.bridgebase.com) has gone from a few thousand people playing at any one time to over 50,000 on most evenings. It has coped with that increase admirably. There has also been an increase in people wanting to have online lessons (I might squeeze in a few more if anyone wants to contact me on [email protected]). But somehow there has been a huge decrease in people sending me their slams. I am sure a lot of you are playing online and are therefore bidding plenty of slams, both good and bad. Please keep sending them in. Send your slam hands to: [email protected]

Page 48 BRIDGE August 2020 bid this slam with her partner Sascha Wernle: Answers to Bernard Magee’s

Dealer North. Game All. Bidding Quizzes 10-12 ♠ 2 ♥ Q 10 9 6 5 ♦ 9 8 5 4 2 on page 7 ♣ 7 4 ♠ Q J 10 8 6 5 4 3 ♠ 9 7 ♥ Void N ♥ K 3 W E ♦ J 10 7 ♦ Q 6 3 S ♣ 10 3 ♣ A Q J 8 6 2 ♠ A K ♥ A J 8 7 4 2 game – the question to ask yourself is ♦ A K 10. Dealer West. Game All. whether partner’s 1NT promises a full ♣ K 9 5 ♠ 6 4 3 ♠ 9 8 spade stopper? ♥ K 8 7 N ♥ A 9 5 4 2 If it does, perhaps you can bid 3NT. W E ♦ K Q 5 3 2 ♦ 7 6 4 S If your partner might have bid it with The bidding was: ♣ A 6 ♣ Q 5 3 a partial stopper, 10-x-x-x or J-x-x for example, you can bid 3♠ – a bid of the West North East South opponents’ suit – to show your strength Pass 1♣ Dbl West North East South and ascertain whether your partner does 4♠ Pass Pass Dbl 1NT Pass 2♦* 2♠ have a stopper. Pass 4NT Pass 5♣ ? Given your strength and poor spade Pass 5♦ Pass 6♥ * transfer to hearts holding, I think you should ask your All Pass partner just in case. On this hand, East would rebid 4♣ over 3♠ and you will end This deal illustrates how well Pass. Your partner has shown five in 5♣. experts deal with high-level bidding hearts so you have to decide whether to Over the 1♠ overcall East wanted to problems. East-West were playing show that you have a fit by bidding 3♥. bid something – he could have chosen a prepared 1♣ opening and South’s When you are vulnerable, you need to double, but 1NT looks reasonable too. first two doubles were for take-out, be careful with weak no-trump hands – if the second one showing significant your partner is weak for his transfer, a extra values. Generally North would 3♥ bid may well take you too high and pass this second double unless she two off (-200) would be a disaster. You 12. Dealer North. Game All. had good distribution (for me that really need four-card support to bid 3♥, ♠ J 4 ♠ K Q 8 6 5 2 especially when you are vulnerable. ♥ 2 N ♥ J 6 3 is either a six-card suit or two five- W E card suits). North’s 4NT showed two The key is to remember that your ♦ A 9 8 6 5 S ♦ K Q 2 places to play, ie at least 5-5. South, partner has another bid and will try to ♣ A K 9 4 2 ♣ 7 thinking North had the minors, compete if he has a suitable hand. Here, bid 5♣ and North bid 5♦ showing he has a poor hand so would let 2♠ lie. the red suits. South’s hand was now 3♥ could easily end up two off. West North East South huge and, with the ♣K protected, 3♥ 3♠ Pass he jumped to 6♥. Well bid. n ?

REDUCE THE COST OF 11. Dealer West. Game All. ♠ ♠ ♠ YOUR POSTAGE 6 5 J 10 4 4 . You have a hand with a lot of ♥ K Q 5 4 N ♥ A 2 potential (two five-card suits with fast W E ♦ A J 2 ♦ K 6 3 winners, a singleton in the opposing suit British postage stamps for sale at S 90% of face-value, all mint and ♣ A K Q 2 ♣ 9 8 7 6 3 and a partial fit) – should you look for a with full gum. Values supplied in fit in a minor? 100s, higher values available, as Much of the time your partner will well as 1st and 2nd class. West North East South have six spades, so you should settle 1♥ 1♠ 1NT 2♠ for spades – it might be wrong, but it is ? easier to make 10 tricks than 11 tricks ( 020 8422 4906 or 07719753767 and the major scores more. 8 [email protected] 4♠ is a good contract, 5♦ might make 3♠. With 19 points you are going for as well, but 620 beats 600. n

BRIDGE August 2020 Page 49 The Diaries of Wendy Wensum Episode 100: Presidential Progress

large number of members had perhaps peevishly, commented ‘An By the time we played this board gathered for the annual general honour to play with you, Madam neither Millie nor I were thinking too meeting of the Riverside Bridge President.’ ‘Yes, it must be,’ she replied analytically. AClub where one agenda item was the without hint of irony, still raising her election of a loyal and upright member glass in response to the greetings of to serve as club president for the year club members as they made their way Dealer North. Love All. ahead. Some uncharitable members from bar to playing area. ♠ A 8 had suggested that the vetting process As we progressed around the room ♥ K 8 7 4 and criteria for potential holders during play, our opponents plied ♦ 10 9 7 6 3 2 of this post had become very lax in us with drinks in celebration of her ♣ 10 recent times. Nevertheless, Millie was election to high office. The wine was ♠ K Q 9 7 5 4 ♠ J 6 3 unanimously voted into this august certainly flowing. Millie’s acceptance ♥ Q J N ♥ 10 6 W E ♦ 5 ♦ A K J and distinguished office. of the adulation reminded me of the S Congratulated on her new historic accounts of the first Queen ♣ Q 7 5 2 ♣ K J 9 8 3 appointment by the assembled Elizabeth’s Progress with pomp and ♠ 10 2 company, she raised one hand and gave circumstance through the city of ♥ A 9 5 3 2 a sublime royal wave of acceptance in Norwich many centuries ago, waving ♦ Q 8 4 acknowledgement of her elevation. to the pheasants and feasting on ♣ A 6 4 With a glass of brandy in the other peasants. hand she toasted her fellow members Back in the present, my recent with grateful aplomb. She was in her resolution to cut down on alcohol Our opponents were Ray and Janet. As element and the gossip that Titan, her consumption was well and truly we approached their table I mentioned cat, had got drunk again the other broken. In the current circumstances, to Millie that in his youth during week was clearly not on her conscience. trying to handle a star-struck Millie, I the sixties Ray was a great fan of the Before bridge commenced, I jokingly, felt this to be perfectly justified. Stones, adding that he still enjoyed going on his motorbike to rock and jazz festivals, a likely cause of his partial deafness. ‘According to his wife, he still plays his trombone at home,’ Millie noted in a superior tone, continuing with ‘No wonder she sends him out to play bridge in the evening with her sister, Janet.’ I couldn’t help thinking that the remark did not seem very presidential. Millie passed as dealer and Janet opened 1♣. I overcalled 1♥ after which Ray jumped to 2♠. Millie’s exuberant 4♥ ended the auction.

West North East South Ray Millie Janet Wendy Pass 1♣ 1♥ 2♠ 4♥ All Pass

Page 50 BRIDGE August 2020 The Diaries of Wendy Wensum Answers to Sally Brock’s Episode 100: Simple Take-Out Doubles Presidential Progress Quiz, Part 2 on page 18

What do you bid, as West, on the Hand 1 With both majors you should following hands with the auctions cue-bid 2♦. Partner will bid given? The vulnerability and scoring a 4-card major if he has one and you will raise him to method should not affect your game. If he rebids 2NT then answers. he denies a 4-card major and Ray selected his opening lead carefully you can bid 3NT. If he bids and loudly asked ‘Any questions 3♣, then all you can do is bid partner?’ Hand 1 Hand 2 3♦ – he will bid 3NT with a He is the only person I know ♠ K Q 5 4 ♠ 5 4 diamond stopper, or else a who can shout into both one’s ears ♥ A J 10 2 ♥ A 3 2 chunky 3-card major and you can let him play in a 4-3 fit. simultaneously. On the few occasions ♦ 8 7 ♦ K 5 Hand 2 It might seem that a jump ♣ Q 4 3 ♣ A Q J 10 5 4 he achieves a good score, everybody in to 3♣ would be the answer the room soon knows about it. but you are much too strong His partner, Janet, timidly answered, for that. Start with a cue-bid ‘No, thank you,’ in a hushed whisper as West North East South of 2♦. Partner will surely bid if compensating for Ray’s thunderous 1♦ Dbl Pass a major and then you can voice. ? bid 3♣, which is forcing. The ♣2 appeared on the table. I On the next round you will bid 3NT showing a single won in hand with the ace, and ruffed diamond stopper, and a club. A small trump to the ace in Hand 3 Hand 4 partner can choose between hand was followed by another club ♠ A 10 9 4 ♠ 9 8 7 3 playing in 3NT and 5♣. ruff. I removed the remaining trumps ♥ K 4 ♥ K Q J 10 5 4 Hand 3 Although you have a decent when I cashed dummy’s ♥K as the suit ♦ Q J 10 4 ♦ K 5 4-card diamond suit, your luckily broke evenly. I then led a small ♣ J 5 3 ♣ 4 best chance of game is diamond from dummy and Janet in no-trumps. Bid 2NT to show your 11 HCP. popped up with the ace and played the Hand 4 Although you have only 9 West North East South king before switching to a spade. The HCP, you have excellent ♦Q, together with the ♠A, gave me the 1♠ Dbl Pass distribution especially if contract. ? partner is short in spades ‘Well bid Madam President’, as you expect. Don’t give Ray bawled at Millie in the style of him the chance to go wrong the hair-drier treatment allegedly Hand 5 Hand 6 – jump straight to 4♥. Hand 5 You have support for partner handed out to wayward footballers ♠ A Q 10 ♠ 5 but the most likely game is ♥ K 10 3 ♥ K Q J 10 3 by some managers. The hand was a in no-trumps, so bid 2NT to triumph for the new president and ♦ A J 4 ♦ A J 4 show 18-20 HCP balanced that evening she could do no wrong. ♣ A J 10 4 ♣ A K 3 2 and leave the rest to partner. It seemed inappropriate to point out Hand 6 Again, you have support that the contract can be defeated. The for partner but he has not opposition needs to find a diamond Hand 7 Hand 8 promised much of a suit. lead to the king, followed by the return ♠ 5 ♠ 5 4 If game is on, it is most likely in hearts, so bid ♦ ♥ A Q 4 3 ♥ A K 3 2 of the J for a ruff. Now as long as a 2♥ and see whether he ♦ ♦ spade switch comes next to set up a A J 4 3 A K 6 2 can show some values. spade winner then, together with the ♣ K Q 5 4 ♣ A K 5 Hand 7 A simple raise to 3♣ is ♦A, the contract will fail by one trick. enough on these cards. As the session ended Ray shouted 5♣ is a long way away. from across the room, ‘Enjoy your West North East South Hand 8 With lots of points and no term of office, Madam President.’ With 1♠ spade stopper, cue-bid their suit (2♠) and hope Millie as president, life at the Riverside Dbl Pass 2♣ Pass partner can bid no-trumps. will certainly not be dull over the next ? twelve months. n

BRIDGE August 2020 Page 51 Mr Bridge Limited Edition Fine Bone China Mugs

£25 per set of four fine bone china mugs Price includes delivery to UK mainland addresses ( 01483 489961